82
系统解剖学习题(留学生) 1 Problem sets on Systematic Anatomy 《系统解剖学》 习题集 Dept. of Anatomy Guilin Medical College

Mid term exam(anatomy) 2012

Embed Size (px)

DESCRIPTION

it is a file that help MBBS 1st year students for prepare 1st exam ....

Citation preview

Page 1: Mid term exam(anatomy) 2012

系统解剖学习题(留学生) 1

Problem sets on Systematic Anatomy

《系统解剖学》

习题集

Dept. of Anatomy

Guilin Medical College

Page 2: Mid term exam(anatomy) 2012

系统解剖学习题(留学生) 2

骨 gǔ

学 xué

、 关 guān

节 jié

学 xué

Osteology and synosteology

1.A Joint united by dense fibrous tissue that permits a slight degree of movement is a: A. Suture B.Syndesmosis C. Symphysis D. Synchondrosis E. Synostosis 2.The intervertebral foramen is surrounded by the: A.Vertebral boby and vertebral arch B.Vertebral boby and pedicle of vertebral arch C.Vertebral boby and lamina of vertebral arch D.Pedicle and lamina of vertebral arch E. Superior and inferior notches of each contiguous pair of vertebrae 3. A joint of the body that contains a broad, fIat disc of fibrocartilage would be classified as a: A. Ball and socket joint B. Suture C. Symphysis joint D. Gliding joint E. Hinge joint 4.The sternal angle articulates on either side with the : A .First costal cartilage B.Sencond costal cartilage C.Third costal cartilage D.Fourth costal cartilage E.Sixth costal cartilage 5.The highest point of the crest of the ilium is on a level passing through the : A .First lumbar spinous process B.Sencond lumbar spinous process C.Third lumbar spinous process D.Fourth lumbar spinous process E.Fifth lumbar spinous process 6.The ligaments, which connect the laminae of adjacent vertebrae, are the : A. Anterior lonlongitudinal ligament B. Posterior lonlongitudinal ligament C. Ligmenta flava D. Supraspinal ligaments E. Interspinal ligaments 7.The should joint is capable of : A. Flexion and extension B. Adduction and abduction

Page 3: Mid term exam(anatomy) 2012

系统解剖学习题(留学生) 3

C. Rotation D. Circumduction E. All movement of the above 8.The movements of pronation and supination of forearm take place at the : A .A elbow joint B.Radio­carpal or wrist joint C.Elbow and wrist joint D.Radio­ulnar joints E.Radio­ulnar and wrist joints 9.Which movements are possible at both the shoulder and elbow joint : A .Abduction and flexion B.Circumduction and flexion C.Circumduction and extension D.Extention and flexion E.Rotation and flexion 10.Which types of joint provide triaxial movement ? A. Hinge B. Ball and socket C. Saddle D. Condyloid E. None do 11. When standing in the Anatomical Position the palms of the hands face : palms of the han< A. Anteriorly B. Laterally C. Medially D. Posteriorly E. Superiorly 12. After ramming the point of his shoulder into apractice dummy, a football player suffered a severe shoulder separation. Although this is a dislocation of the acromioclavicular joint, several structures could be torn, including the one that gives the joint its greatest strength and stability, namely the : A. Acromioclavicular ligament B. Coracoacromial ligament C. Coracoclavicular ligament D. Supraspinatus tendon E. Tendon of the long head of biceps 13. While performing an arthrogram to study an apparent rotator cuff injury, it was noted that the contrast material had spread from the shoulder joint onto the anterior lateral surface of the scapula near the joint. When asked, the first year resident responded that this was due to an anterior tear in the cuff. Having just studied the shoulder joint you respond that the contrast is in is a normal extension of the joint cavity called the: A Bicipital bursa B. Olecranon bursa C. Subacromial bursa

Page 4: Mid term exam(anatomy) 2012

系统解剖学习题(留学生) 4

D. Subscapular bursa E. Ulnar bursa 14. The axis of rotation ( pronation /supination ) at the distal radioulnar joint is through the: A Capitate bone B. Head of tha radius C. Head of the ulna D. Styloid process of the radius E. Styloid process of the ulna 15. A two­year­old child will not go to take her nap. Her mother tightly holds her left hand as sheleads her to the bedroom. Refusing to go further, the child suddenly attempts to jerk awayand then sits down screaming and holding her left elbow. In a attempt to calm her down hermother offers her a cookie, but she cannot supinate her left hand to receive it. Which joint wasdislocated? A The glenohumeral joint B. The humero­ulnar joint C. The humero­radial joint D. The proximal radio­ulnar joint E. The distal radio­ulnar joint 16. You are in the emergency room when a student is brought in with a shoulder injury sustainedwhile playing touch football. In comparing the symmetry of his two shoulders, you notice amarked elevation of the distal end of his clavicle with respect to the acromion on the injured side.X­ray exam reveals a grade HI shoulder separation. In order for this to have occurred, whichligament must be torn? A. Coracoacromial B. Coracoclavicular C. Costoclavicular D. Superior glenohumeral E. Transverse humeral 17. A 3­year­old child walking hand­in­hand with her father screams in pain as he jerks her quickly up onto the curb to dodge a speeding car. The examining physician calls it a case of "pulled elbow",a dislocation sometimes seen in young children and caused by:; A. The head of the radius slipping part way out of the annular ligament B. Tear of the common extensor tendon C.. Stretching of the radioulnar interosseous membrane D. Tear of the ulnar collateral ligament E. radial collateral ligament 18. It was determined that a football player tore his coracoclavicular ligament. This is an, example of a: A. Pulled elbow B. Rotator cuff tear C. Separated shoulder D. Dislocated shoulder E. Colles fracture 19. While walking to class on an icy winter morning, a student slips and falls on her outstretchedhand. The intense pain forces her to go td the emergency room. After X­rays of her

Page 5: Mid term exam(anatomy) 2012

系统解剖学习题(留学生) 5

wrist aretaken, the attending says, " You were lucky, there is no Colles'nor scaphoid fractures, but you have dislocated the middle carpal bone of the proximal row. " Which bone was dislocated? A. Capitate B. Lunate C.Scaphoid D.Trapezoid E. Triquetrum 20. The fibrocartilagious structure which deepens the shoulder socket is the: A.Articular capsule B. Articular cartiIage C. Glenoid labrum D. Lateral meniscus E. Superior glenohumeral ligament 21. The synovial cavity of the glenohumeral joint communicates with the subdeltoid (subacromial )Bursa after the rupture of the: A. Infraspinatus tendon B. Middle glenohumeral ligament C. SubScapularis tendon D. Supraspinatus tendon E. Long head of the biceps brachii tendon 22. The interosseous membrane between the radius and ulna is the uniting structure in a type of fibrous joint classified as a : A. Suture B. Synphysis C. Synchondrosis D. Syndesmosis E. Synostosis 23. An example of a temporary cartilaginous joint is a/an: A. Articular disk B. Epiphyseal plate or growth plate C. Intervertebral disk D. Meniscus E. Primary ossification center 24.Which jointwouldbe subject to synovitis (inflammation of the synovial membrane) ? A. Epiphyseal plate B. Metacarpophalangeal C. Pubic symphysis D. Radioulnar E. Suture 25.Which of the following structures is unique to a synovial joint? A.Accessory ligaments B.Bursae C.Collateral lIgamnents D.Fibrocartilage E.Joint cavity

Page 6: Mid term exam(anatomy) 2012

系统解剖学习题(留学生) 6

26.In an autoaccident, thepatient′s knee strikes the dashboard which in turn pushes the head of thefemurposteriorly out of its socket. Which ligament is most likely ruptured by this posterior dislocation? A. I1iofermoal B. Ischiofermoal C. P ubofermoal D. Transverse acetabular E. All of above 27. To test the integrity of the knee joint, a physician pulls anteriorly on the flexed leg of his patient. This “drawer” test is positive if the leg moves excessively anteriorward. This would indicate a weakness in or rupture of the : A. Medial meniscus B∙ Posterior cruciate ligament C∙ Fibular collateral ligament D∙ Medial collateraI ligament E. Anterior cruciate ligament 28. One of the menisci of the knee is often injured in a sprain of the knee because the : A.Anterior cruciate ligament is attached to the lateral meniscus B.Anterior cruciate ligament is attached to the medial meniscus C.Lateral collateral ligament is attached to the lateral meniscus D.Medial collateral ligament is attached to the mediall meniscus E.Posterior cruciate ligament is attached to the lateral meniscus 29.A soldier developed”fallen arches” from marching with a heavy pack in boots that lacked arch support . The ligament that normally supports the head of the talus and is primarily responsible for holding up the medial longitudinal arch of the foot is the : A.Calcaneometatarsal B.Deltoid C.Long plantar, D.Plantar calcaneonavicular (spring) E.Short plantar 30. A player is blocked from behind, during a kick­off return, injuring his medial collateral liga­ment. The team doctor tests his knee by pulling anteriorly on the leg with the knee flexed. If theleg translates (moves) forward significantly, this indicates damage to which structure? A. Anterior cruciate ligament B. Lateral collateral ligament C. Medial meniscus D. Medial collateral ligament E. Posterior cruciate ligament 31. During a basketball game, the center of the team went up for a rebound and when coming down, her foot landed da the foot of another';player, sharply everting it. She limped off the floor,having severely sprained the medial side of her ankle. Which ligament was injured? A. Calcaneofibular B. Deltoid C. Short plantar ligament D. Plantar calcaneonavicular

Page 7: Mid term exam(anatomy) 2012

系统解剖学习题(留学生) 7

E. Tibial collateral ligament 32. Which ligament limits extension at the hip joint? A. Iliofermoral B. Ligament capitis femorus C. Pubofemoral D. Zona orbicularis E. All of above 33. In in]uries of the knee, :the medial meniscis is frequently torn because it is firmly attached to which struclure? A. Anterior cruciate Iigamlent B. Fibular collateral ligament C. Tibial collateral ligamlen D. Patellar ligament E. Patellar retinaculum 34. At the temporomandibular joint (TMJ) , hinge movements occur between the: A. Condyle and articular eminence B. Articular disk and articular eminence C. Condyle and articular disk D. Articular disk and articular cavity E. Condyle and articular cavity 35. The emiddle nasal concha is a part of whish bone? A. Ethmoid B. Maxillary C. Palatine D. Sphenoid E. All of the above

Answer: 1.B 2.A 3.C 4.B 5.D 6.C 7.E 8.D 9.D 10.B 11.A 12.C 13.D 14.E 15.D 16.B 17.A 18.C 19.B 20.C 21.D 22.D 23.B 24.B 25.E 26.B 27.E 28.D 29.D 30.A 31.B 32.A 33.C 34.C 35.A

Page 8: Mid term exam(anatomy) 2012

系统解剖学习题(留学生) 8

肌 jī

学 xué

Myology

1. The connective tissue component of a skeletal muscle that surrounds fasciculi is called the : A. Perimysium B. Epimysium C. Endomysium D. Tendomysium E. Superficial fascia 2. The trapezius muscle is named on the basis of : A. Shape B. Size C. Location D. Action E. Orientation 3. Which muscle is involved in moving the vertebral column? A. Scalene B. Sacrospinalis C. Sartorius D. Diaphragm E. A and B 4. A muscle that originates on both the spine of the scapula and the clavicle is the: A.Pectoralis major B.Trapezius C.Deltoid D.Stemocleidomastoid E.Erector spine 5. All of the following muscles are both the adductor and medial rotator of the arm except the: A. Pectoralis major B. Infraspinatus C. Teres major D. Subscapularis E. latissimus dorsi 6. Which muscle contributes to the "rotator cuff"? A. Deltoid B. Latissimus dorsi C. Pectoralis minor D. Supraspinatus E. Teres major 7. A patient presented to his physician with chronic shoulder pain. It was noted that when asked toabduct his arm, he initially leaned laterally?and then straightened up. When iodinated contrastwas injected into his shoulder joint it was found to be in the subdeltoid bursa as well as in the joint

Page 9: Mid term exam(anatomy) 2012

系统解剖学习题(留学生) 9

cavity. Which structrure was damaged to produce the shoulder pain? A. Acromioclavicular ligament B. Long head of the biceps braehii muscle C. Subscapularis muscle D. Superior glenohumeral ligament E. Supraspinatus muscle 8. Because the biceps braehii muscle flexes the forearm when it­contracts; most of the muscle lies. A. Anterior to the humerus B. Posterior to the humerus C. Anterior to the ulna and radius D. Posterior to the ulna and radius E. Superior to the ulna and radius 9. One of the motor components of the cervical plexus, the ansa cervicalis, innervates all of the following muscles except the: A. Omohyoid B. Sternohyoid ': C. Stemothyroid D. Stylohyoid E. Thyrohyoid 10. A six­year­old child, whose medical history includes a rather difficult birth, has a permanently tilted head posture, with the right ear near the right shoulder and the face turned upward and tothe left. Which of the following muscles was very likely damaged during birth? A Anterior scalene B. Omohyoid C. Sternocleidomastoid D. Trapezius E. Platysma 11. Which of the following hyoid muscles is an important landmark in both the anterior and poste­rior triangles of the neck? A. Geniohyoid B. Mylohyoid C. Omohyoid D. Sternohyoid E. Stylohyoid 12. An elderly man complained of pain in his shoulder when he brought his forearm and hand be­ hind his back while dressing. It Was determined that stretching of the lateral rotators of his armduring­this motion caused the pain. Which muscle was most likely involved? A. Infraspinatus B. Latissimus dorsi C. Subscapularis D. Supraspinatus E. Teres major 13. What muscle tendon is enclosed within its own synovial sheath in the carpal canal? A. Flexor carpi ulnaris

Page 10: Mid term exam(anatomy) 2012

系统解剖学习题(留学生) 10

B. Flexor digitorum profundus to 2nd digit C. Flexor digitorum superficialis to 2nd digit D. Flexor pollicis longus E. Palmaris longus 14. After falling on the ice, it was determined that a patient had a Colles' fracture?Care'must betaken to relieve tension on the broken distal end of the radius created by the pull of which muscle? A. Extensor carpi ulnaris B. Brachioradialis C. Extensor carpi radialisl longus D. Pronator quadratus E. Extensor carpi radialisl brevis 15. lf the musculocutaneous nerve issevered at its orgin from the brachial plexus , flexion at the elbow is greatly weakened but not abolished. What muscle remains operative and can contribute to flexion? A. Brachialis B. Brachioradialis CCoracobrachialis D. Long head of biceps brachii E. Short head of biceps brachii 16. Development of “tennis elbow"(lateral epicondylitis) involves the origin of which muscle? A. Abductor pollicis longus B. Anconeus C. Brachioradialis D. Extensor carpi radialis brevis E. Triceps brachii 17. Several deficits in muscle functbon of the right upper limb were noted,including inabillty to abduct the arm .This was caused by denervation of which muscle? A. Deltoid B. Infraspinatus C. Latissimus dorsi D. Teres minor E. Trapezius 18. Later, while undergoing physical therapy because of his shoulder injury, he comments that it is very painful when his forearm is brought across his chest (medial or internal rotation ofthe humerus) . You deduce that the pain is due to stretching of the lateral (external) rotators ofthe shoulder. Which muscle was most likely the source of his pain? A.Infraspinatus B.Latissimus dorsi C.Rhomboideus major D.Supraspinatus E.Teres major 19. An elderly patient complains of shoulder and has difficulty abducting his arm.Arthroscopy is done in which a dye is injected into the shoulder joint and an X­ray taken. The radiologist

Page 11: Mid term exam(anatomy) 2012

系统解剖学习题(留学生) 11

notes that the dye has leaked from the shoulder joint into the subacromial bursa. What tendon would need to be ruptured for this to occer? A. Deltoid B. Infraspinatus C. Latissimus dorsi D. Supraspinatus E. Teres minor F. Latissimus dorsi 20. Which movement would fail in case of paralysis of the quadriceps femoris muscle? A. Adduction at the hip B. Extension at the hip C. Extension at the knee D. Flexion at the knee E. Medial rotation at the knee 21. Following a penetrating injury to the left femoral triangle, a patient related that walking was virtually impossible because at every step the left knee collapsed into flexion. This history suggests paralysis of which muscle? A. Adductor magnus B. Biceps femoris Q Gluteus maximus D. Quadriceps femoris E. Sartorius 22. An obturator hernia that compresses the obturator nerve in the obturator canal may affectfunction of all of the following muscles EXCEPT: A. Adductor brevis B. Adductor longus C. Gracilis D. Obturator'eKtemus E. Pectineus 23. What muscle passes through the lesser sciatic foramen? A. Gluteus minimus B. Obturator internus C. Piriformis D. Quadratus femoris E. Superior gemellus 24. When, in approximately 12%, the common fibular nerve passes through the piriformis muscle, the nerve may be compressed. This would affect part of which muscle? A. Adductor magnus B. Biceps femoris C. Gluteus maximus D. Semimembranosis E. Semitendinosis 25. A patient with painful swelling in the distal calf cannot plantar flex at the ankle with any power. Which tendon was likely ruptured? A. Calcaneal

Page 12: Mid term exam(anatomy) 2012

系统解剖学习题(留学生) 12

B. Extensor digitorum longus C. Extensor hallucis longus D. Plantaris E. Tibialis 26. During a strenuous game of tennis a 55­year­old woman complained of severe shoulder pain that forced her to quit the game. During physical examination it was found that she could not initiate abduction of her arm, but if her arm was elevated to 45 degrees from the vertical (at her side ) position, she had no trouble fully abducting it. Which muscle was injured responsible? A. Deltoid B. Infraspinatus C. Supraspinatus D. Teres major E. Trapezius 27. The rotator cuff is composed of all the following muscles EXCEPT: A. Infraspinatus B. Subscapula C. Supraspinatus D. Teres major E. Teres minor 28. Which muscle is the strongest medial rotator of the arm? A. Coracobrachialis B. Intraspinatus C. Subscapularis D. Supraspinatus E. Teres minor 29. In the lumbar region, tuberculosis may to spread from the vertebrae into an adjacent muscle to produce an abscess. Pus from the abscess may travel within the fascial sheath surrounding the affected muscle. A patient presents with pus surfacing in the superomedial part of the thigh. To which muscle did the tuberculosis most likely spread ? A. Internal oblique B. Obturator internus C. Psoas major D.Quadratus lumborus E. Rectus abdominus 30. After being thrown from a motorcycle moving at high spread, a 16­ year­old female was found to have a paralyzed right pectoralis major muscle. Which set of movements at the shoulder joint would be found greatly weakened? A. Abduction and extension B. Abduction and lateral rotation Q Adduction and flexion D. Lateral rotation and extension E. All of the above

Page 13: Mid term exam(anatomy) 2012

系统解剖学习题(留学生) 13

Answer: 1.A 2.A 3.E 4.C 5.B 6.D 7.E 8.A 9.D 10.C 11.C 12.A 13.D 14.B 15.A 16.D 17.A 18.A 19.D 20.C 21.D 22.E 23.B 24.B 25.A 26.C 27.D 28.C29.C 30.C

Define the following Terms 1) Anatomical position 2) Sternal angle 3) Costal arch 4) Pterion 5) Vertebral column 6).thoracic cage 7) Intervertebral discs 8) Pelvis 9) Scalene fissure

Essay question 1) Describe the structures of bone from superficial to deep . 2) Describe the general features of the vertebrae and the main characteristics of the vertebrae in each region. 3) Describe the essential structures of synovial joints. 4) Describe the composition,the structures and the movements of shoulder joint. 5) Describe the composition,the structures and the movements of elbow joint. 6) Describe the composition,the structures and the movements of knee joint. 7) Please compare hip joint with shoulder joint based on the essential structures, accessory structures and the functions.. 8)Describe the composition and the movements of wrist joint and ankle joint.

Page 14: Mid term exam(anatomy) 2012

系统解剖学习题(留学生) 14

消 xiao

化 huà

系 xì

统 tǒng

Digestive system

1. The terminal (end) portion of the small intestine is the: A. Ileum B. Cecum C. Duodenum D. Jejunum E. Ascending colon 2. The bulk of a tooth is composed of: A. Dentine B. Enamel C. Cement D. Dental pulp E. Alveolar bone 3. Which of the following is not considered part of the digestive system? A. Pancreas B. Spleen C. Tongue D. Cecum E. Vermiform appendix 4. The palatine tonsil is placed at the lateral wall of the: A. Nasopharynx B. Oropharynx C. Laryngopharynx D. Oral cavity E. Laryngeal cavity 5. Fylonc stenosis will interfere most directly with the passage of materials from the: A. Esophagus into the stomach B. Pharynx into the esophagus C. Ileum into the cecmn D. Stomach into the duodenum E. Duodenum into the cecum 6. Which of the following statements concerning the stomach is false. A. It is the most dilated part of the digestive tube B. Lies in the left hypochondriac and epigastric regions of the abdomen C. The cardiac orifice communicates with the esophagus D. The pyloric orifice communicates with the duodenum E. Consists of the fundus and body of the stomach 7. Mesentery, lesser omentum, and greater ompntum are all directly associated with the : A. Peritoneum B. Liver C. Esophagus

Page 15: Mid term exam(anatomy) 2012

系统解剖学习题(留学生) 15

D. Mucosa of the alimentary cana E. Common bile duct 8. If an incision had to be made in the small intestine to remove an obstruction­ which layer of tissue would be cut first? A. Muscularis B. Mucosa C. Serosa D. Submucosa E. None of these 9. Which of the following structures could not be seen when looking directly into the oral cavity? A. Dorsum of the tongue B. Uvula C. Palatine tonsil D. Epiglottis E. Papilla of parotid duct 10. Which of the following is NOT found within the hepatoduodenal ligament? A. Portal vein B. Fibroblasts C. Main pancreatic duct D. Proper hepatic artery E. Postganglionic sympathetic nerve fibers 11. The stomach: A. Is the first part of the alimentary canal B. Has a smooth interior surface lined by stratified squamous epithelium C. Receives the esophagus at its fundus D. Is normally covered by peritoneum on its posterior surface E. Lies anterior to both the omental bursa and the greater omentum 12. In a surgical approach to the stomach, upon opening the abdominal cavity, what peritoneal specialization (s) would you expect to encounter in the upper right quadrant? A. Median umbilical ligament B. Falciform ligament C. Conjoint tendon D. Greater omentum E. Both B and D are correct 13. The liver: A­ Usually cannot be palpated below the costal margin in a healthy person B. Is an exocrine gland that secretes erythropoietin C. Has a porta hepatis from which the common bile duct exits D. Is innervated by the vagus nerve, but lacks a sympathetic nerve supply E. Has superficial and deep lymphatics, some of which drain toward the diaphragm and thorax 14­ The gall bladder: A. When inflamed or distended may refer pain to the right shoulder region B. Is lodged in the porta hepatis of the liver C. Is retroperitoneal D. Is an exocrine gland that produces and secrets bile

Page 16: Mid term exam(anatomy) 2012

系统解剖学习题(留学生) 16

E. Contacts the anterior abdominal wall on the left side of the falciform ligament 15. The celiac trunk: A. Normally has four main branches B. Usually is at the level of the esophageal hiatus C. Is located in the splenorenal ligament D. Has a branch, the splenic artery that supplies the body and tail of the panaceas E. Has branches which usually supply the suprarenal glands 16. Which of the following statements about the portal system is FALSE? A. Its tributaries lack valves B. It begins in capillaries and ends in hepatic sinusoids ? C. It usually drains only the alimentary canal D. It carries deoxygenated blood E. It has clinically important anatomoses with the systemic venous system 17. Which of the following is TRUE concerning the duodenum? A. It is completely retroperitoneal B. Its 2nd part is\anterior to the hepatic flexure C. Its 3rd part is anterior to the path of the superior mesenteric artery D. Its 3rd part crosses anterior to the body of the first lumbar vertebra E. Its junction with the jejunum is stabilized by a fibromuscular band 18. The inferior mesenteric artery: A. Has only retroperitoneal branches B. Carries a plexus of vagal nerve fibers along its surface C. Branches from the aorta posterior to the neck of the pancreas D. Has a right colic branch that courses anterior to the inferior vena cava E. Gives rise to sigmoidal and superior rectal arteries 19. The large intestine: A. Has a right (hepatic) flexure that is normally higher than the left (splenic) flexure B. Has a mesentery suspending only the transverse colon C. Can be found in all four abdominal quadrants D. Is longer than the small intestine E. Has numerous plicae on its inner surface 20. The appendix: A. Lacks an outer longitudinal muscle layer B. Lies in the retroperitoneal space at McBurneys point C. Has lymphatic drainage into the inferior mesenteric nodes D. Typically receives its blood supply from an intestinal branch of the superior mesenteric artery E. Has its own mesentery and its own artery 21. One way to distinguish the jejunum from the ileum is that: A. Its external surface is characterized by haustra B. It contains fewer epiploic appendages than the ileum C. It lies in the left upper quadrant D. It lies intraperitoneal while the ileum lies retroperitoneal E. It is lined by stratified squamous epithelium 22. The blood supply to the gastrointestinal organs: A. Is constant in its pattern

Page 17: Mid term exam(anatomy) 2012

系统解剖学习题(留学生) 17

B. In the adult, always courses between layers of mesentery C. Rarely contains vasa recta D. Commonly form anastomotic connections E. All of the above 23. The pancreas: A. Receives its only arterial supply from the splenic artery B. Has an uncinate process located posterior to the course of the superior mesenteric artery C. Becomes intraperitoneal at the hepatoduodenal ligament D. Has its head in contact with the hilus of the right kidney E. Is usually located at the level of the L4 vertebra 24. The cecum: A. Exhibits a complete longitudinal muscle layer R Has a lumen characterized by the opening of the vermiform appendix C. Is suspended on the transverse mesocolon D. Is in the left iliac fossa E. Receives its main arterial supply from the middle colic artery 25. Which of the following statements about the internal iliac system of arteries is FALSE? A. The posterior division of the internal iliac artery gives rise to parietal branches only B. The anterior division of the internal iliac artery gives rise to the umbilical artery C. The uterine and vaginal arteries may branch from a common trunk D. The internal pudendal artery exits the pelvis by passing through the lesser sciatic foramen E. The inferior vesical artery supplies both the bladder and the prostate gland in the male 26. Which of the following statements about the rectum is FALSE? A. The lower third of the rectum is entirely retroperitoneal B. By digital examination of the rectum in the male, the prostate can be palpated through the anterior wall of the rectum C. The inner surface of the rectum has three transverse rectal folds on the posterior wall D. The muscle layers of the wall of the rectum are under control of the autonomic nervous system E. Veins in the wall of the rectum from a clinically important communication between the portal and caval venous system 27. Concerning the anal sphincters: A. The internal anal sphincter is a thickened portion of the inner longitudinal layer ofsmooth muscle of the anal canal B. The superficial portion of the external anal sphincter is innervated by inferior rectalBranches of the pudendal nerve C. The deep portion of the external anal sphincter is fusiform in structure D. The subcutaneous portion of the external anal sphincter is the largest portion of the ex­ternal anal sphincter E. The puborectalis muscle does not usually contact any of the anal sphincters 28. A patient was diagnosed with bleeding ulcer of the lesser curvature of the stomach. Which artery is most likely involved? A. Gastroduodenal B. Left gastric C. Left gastro­omental (epiploic)

Page 18: Mid term exam(anatomy) 2012

系统解剖学习题(留学生) 18

D. Right gastro­omental (epiploic) E. Short gastrics 29. In order to do a vagotomy (section of vagal nerve trunks) to reduce the secretion of acid by cells of the stomach mucosa in patients with peptic ulcers, one needs to cut the gastric branches and retain vagal innervation to other abdominal organs. Where would a surgeon look for these branches in relation to the stomach? A. Along the gastroepiploic vessels B. Along the greater curvature C. Along the lesser curvature D. In the base of the omental apron E. In the gastrocolic ligament 30. While performing a splenectomy (removal of the spleen) following an automobile accident, thesurgeons were especially attentive to locate and preserve the tail of the pancreas, which is closelyassociated with the spleen. This they found in the. A. Gastrocolic ligament B. Gastrospenic ligament C. Phrenicocolic ligament D. Splenorenal ligament E. Transverse mesocolon 31. Which of the following structures does not lie at least partially in the retroperitoneum? A. Adrenal gland B. Duodenum C. Kidney D. Pancreas E. Spleen 32. A 60­year­old male executive who had a his of a chronic duodenal ulcer was admitted to theER exhibiting signs of a severe internal hemorhage. He was quickly diagnosed with perforationof the posterior wall of the first part of duodenum and erosion of an artery behind it by the gastric expel. The artery is most likely the: A. Common hepatic B. Gastroduodenal C. Left gastric D. Proper hepatic E. Superior mesenteric 33. An ulcer near the pyloroduodenal junction perforated and eroded a large artery immediately posterior to the duodenum. The ligation of the eroded vessel at its origin would LEAST affectarterial supply to the: A. First part of the duodenum B. Second part of the duodenum C. Greater curvature of the stomach D. Head of the pancreas E. Tail of the pancreas 34. You are observing an operation to remove the left suprarenal gland. To expose the gland. To expose the gland the surgeon mobilizes the descending colon by cutting along its lateral attachment to the body wall and dissecting medialward in the fusion fascia behind it. Suddenly

Page 19: Mid term exam(anatomy) 2012

系统解剖学习题(留学生) 19

the operative field is filled with blood. The surgeon realizes he has failed to cut a mesenteric attachment between the left colic flexure and another organ. As a result of the traction, the surface of the organ tore. Which organ was injured? A. Duodenum B. Kidney C. Liver D. Spleen E. Suprarenal gland 35. A patient was admitted with symptoms of an upper bowel obstruction. Upon CT examination, it was found that the third (transverse) portion of the duodenum was compressed by a large vessel causing the obstruction. The vessel involved is most likely to be the : A. Inferior mesenteric artery B. Superior mesenteric artery C. Inferior mesenteric vein D. Portal vein E. Splenic vein 36. During emergency surgery, it was found that a chronic gastric ulcer had perforated the posterior wall of the stomach and eroded a large artery running immediately posterior to the stomach. The artery is the : A. Gastroduodenal B. Common hepatic C. Left gastroepiploic D. Splenic E. Superior mesenteric 37. Regarding the 2nd portion of the duodenum, all are correct EXCEPT: A. It is crossed by the transverse colon B. It is thin walled and circular folds are absent in is interior C. It has the opening for the common bile duct and pancreatic duct on its posteromedial wall D. It is secondarily retroperitoneal E. Both the gastroduodenal and superior mesenteric arteries supply it 38. In order to approach the area posterior to the stomach, a surgeon decided to go through the lesser omentum. Before incising the mesentery she was careful to find and preserve a nerve lying in the upper portion of the hepatogastric ligament, i.e., the : A. Celiac branch of the anterior vagal trunk B. Celiac branch of the posterior vagal trunk C. Greater splanchnic branch to the right suprarenal gland D. Hepatic branch of the anterior vagal trunk E. Hepatic branch of the posterior vagal trunk 39. Which of the following is NOT in contact with the spleen? A. Colon B. Diaphragm C. Duodenum D. Pancreas E. Stomach

Page 20: Mid term exam(anatomy) 2012

系统解剖学习题(留学生) 20

40. The fundus of the stomach receives its arterial supply from the: A. Common hepatic B. Inferior phrenic C. Left gastroepiploic D. Right gastric E. Splenic 41. During an emergency splenectomy, the surgeon accidentally tore the gastrosplenic ligament and its contents. The artery (ies) likely to be damaged in this event is (are) the: A. Left gastric B. Splenic C. Short gastric D. Middle colic E. Caudal pancreatic 42. While performing emergency surgery to control hemorrhage brought on by arterial erosioncaused by a duodenal ulcer, surgeons ligated the badly .damaged gastroduodenal artery near its origin, which affected all of its branches as well. Assuming "average anatomy", in which of thefollowing arteries would blood now flow in retrograde fashion (backwards) from collateralsources? A, Left hepatic B. Right gastroepiploic C. Short gastric D. Left gastric E. Omental branches 43.A 40 year­old male with a long history of duodenal ulcer problems was brought in for emergency to control severe hemorrhage into the peritoneal cavity. The surgeons found that erosion by the ulcer of a vessel passing behind the first part of the duodenum was the source of the hemorrhage. Which of the following vessels passes behind the first part of the duodenum and would need to be clamped off to control the bleeding? A. Coronary vein B. Gastroduodenal artery C. Inferior pancreatoduodenal artery D. Proper hepatic artery E. Splenic vein 44. During a cholecystectomy ( removal of the gall bladder), the surgical resident accidentally jabbed a sharp instrument into the area immediately posterior to the epiploic foramen (its posterior boundary). He was horrified to see the surgical field immediately fill with blood, the source which he knew was the : A. Aorta B. Inferior vena cava C. Portal vein D. Right renal artery E. Superior mesenteroc vein 45. The division between the true right and left lobes (internal lobes) of the liver may be visualized on the outside of the liver as a plane passing through the : A. gallbladder fossa and round ligament of liver

Page 21: Mid term exam(anatomy) 2012

系统解剖学习题(留学生) 21

B. falciform ligament and ligamentum venosum C. gallbladder fossa and inferior vena cava D. falciform ligament and right hepatic vein E. gallbladder fossa and right triangular ligament 46. Orally ingested contrast medium opening all of the following structures EXCEPT the: A. Colon B. Duodenum C. Esophagus D. Gall bladder E. Stomach 47. To stop hemorrhaging from a ruptured spleen, it was necessary to temporarily ligate the splenic artery near the celiac trunk. Which structure of the blood supply is least likely to be affected by the ligation? A. Duodenum B. Greater omentum C. Body of pancreas D. Tail of pancreas E. Stomach 48. A 50­year­old female patient with severe jaundice was diagnosed with pancreatic cancer­ You suspect that the tumor is located in­which portion of the pancreas? A. Head B. Neck C. Body D. Tail E. Uncinate process 49. A patient was admitted with symptoms of bowel obstruction. Further examination revealed that the obstruction was caused by the nutcracker like compression of the bowel between the superior mesenteric artery and the aorta. The compressed bowel is most likely the: A. Duodenum B. Jejunum C. Ileum D. Ascending colon E Transverse colon 50. You are observing a laparoscope cholecystectomy. The surgeon states that he is next going to expose the cystic artery in order to staple across it. He asks you where he should look for it. You reply, "In the triangle of Calot” . What structures form this triangle and are the keys to finding the artery? A. Common hepatic duct, liver and cystic duct B. Cystic duct, right hepatic artery and right hepatic duct C. Gall bladder­ liver and common bile duct D. Left hepatic duct, liver and cystic duct E. Right branch of portal vein, liver and common bile duct

Page 22: Mid term exam(anatomy) 2012

系统解剖学习题(留学生) 22

Answer 1.A 2.A 3.B 4.D 5.D 6. E 7.A 8.C 9.D 10.C 11.D 12.E 13.E 14.A 15.D 16.C 17.E 18.E 19.C 20.E 2l.C 22.D 23.B 24.B 25.D 26.C 27.B 28.B 29.C 30.D 31.E 32.B 33.E 34.D 35.B 36.D 37.B 38.D 39.C 40.E 41.C 42.B 43.B 44.B 45.C 46.D 47.A 48.A 49.A 50.A

Page 23: Mid term exam(anatomy) 2012

系统解剖学习题(留学生) 23

呼 hū

吸 xī

系 xì

统 tǒng

Respiratory system

1. The lining of the chest cavity is called the: A. Pleural cavity B. Endocardium C. Costal pleura D. Visceral pleura E. Mediastinal pleura 2. Which part of laryngeal cavity is the narrowest part? A. Aperture of larynx B. Rima vestibuli C. Fissure of glottis D. Laryngeal vestibule E. Infraglottic cavity 3. The portion of the pharynx that contains the palatine and lingual tonsils is the: A. Laryngopharynx B. Oropharynx C. Nasopharynx D. Esophageopharynx E. Laryngeal cavity 4. The visceral pleura is known as the: A. Costal pleura B. Diaphragmatic pleura C. Pulmonary Pleura D. Mediastinal pleura E. Cupula of pleura 5. All of the following statements about the nasopharynx are the true EXCEPT: A. It communicates with the nasal cavity via choanac B. The auditory tube opens along the lateral wall C. The inferior border is the epiglottis D. Pharyngeal recesses are found in the lateral walls E. The levator veli palatine muscle forms a fold on the lateral wall 6. Concerning the lateral nasal wall: A. The frontal sinus drains inferior to the middle nasal concha B. The ethmoidal bulla is inferior to the inferior nasal concha C. The maxilIary sinus drains superior to the middle nasal concha D. The middle meatus is superior to the middle nasal concha E. The sphenoethmoidal recess is direclly posterior to the inferior nasal concha 7. When examining the nasal cavity of a patient by spreading the nostril with a speculum, you can confirm that: A. The opening of the nasolacrimal duct is in the atrium B. The opening of the anterior ethmoidal air cells is in the sphenoethmoidal recess

Page 24: Mid term exam(anatomy) 2012

系统解剖学习题(留学生) 24

C. The opentng of the maxillary sinus is in the inferior meatus D. The frontonasal duct drains into the semilunar hiatus E. The choanae are between the vestibule and the atrium 8. The ethmoidal air cells: A. Are normally filled with fluid B. Drain poorly because their duct exits the superior aspect of the air cells C. Are fully formed at the time of birth D. Are separated from the orbit by a thick plate of bone E. Are lined by mucous membrane that is innervated by branches of the nasociliary nerve 9. Which of the following statements about the muscles of the soft palate is FALSE? A. The palatopharyngeus muscle underlies the palatopharyngeal fold B. The salpingopharyngeus muscle is part of the longitudinal muscle layer of the pharynx C. The levator veil palatini is the only muscle of the palate that is innervated by CN V D. The musculus uvulae is located at the boundary between the oral pharynx and the nasal pharynx E. The palatoglossus muscle is innervated by the vagus nerve 10. Which of the following structures does not drain into the region inferior to the middle nasal concha? A. Maxillary sinus B. Anterior ethmoidal air cells C. Middle ethmoidal air cells D. Frontal sinus E. Sphenoid sinus 11. The larynx has: A. A quadrangular membrane which extends from the epiglottis to the tongue B. A vocal ligament which extends from the arytenoid cartilage to the cricoid cartilage C. A vestibular fold located superior to the vocal fold D. An aryepiglottic fold along the upper margin of the conus elasticus E. A thyroepiglottic muscle which courses parallel to the vocal ligament 12. In the larynx: A. The vestibule opens posteriorly into the oropharynx B. The saccule lies medial to the quadrangular membrane C. The glottis lies inferior to the vocal folds D. The vestibule lies superior to the vocal folds E. The infraglottic cavity lies between the vocal and vestibular folds 13. Adduction of the vocal folds is a function of the: A. Posterior cricoarytenoid muscle B. Cricothyroid muscle C. Lateral cricoarytenoid muscle D. Aryepiglottic muscle E. Thyroartenoid muscle 14. Your patient, an 86­year­old female who has been bed­ridden and lying supine for many weeks; has developed a right lung abscess that is draining by gravity into one particular region of the lung. Where is the most likely site of fluid accumulation? A. Apical segment of upper lobe

Page 25: Mid term exam(anatomy) 2012

系统解剖学习题(留学生) 25

B. Lingula C. Lower lobe D. Middle lobe E. Superior segment of lower lobe 15. The pleural space into which lung tissue just above the cardiac notch would tend to expand during deep inspiration is the: A. Anterior mediastinum B. Costodiaphragmatic recess C. Costomediastinal recess D. Cupola E. Pulmonary ligament 16. A 16­year­old male suffered a stab wound in which a knife blade entered immediately superior to the upper edge of the right clavicle near its head. He was in extreme pain. which was interpreted by the physician as a likely indicator of a collapsed lung following disruption of the pleura. If that was true­ what portion of the pleura was most likely cut or torn? A. Costal pleura B. Cupola C. Hilar reflection D. Mediastinal pleura E. Pulmonary ligament 17. During a lung transplant procedure, a doctor attempted to pass his index finger posteriorly inferior to the root of the left lung, hut he found passage of the finger blocked. Which structurewould most likely be responsible for this? A. Costodiaphragmatic recess B. Cupola C. Inferior vena cava D. Left pulmonary vein E. Pulmonary ligament 18. Which feature is found only in the left lung? A. Cardiac notch B. Horizontal fissure C. Oblique fissure D. Superior lobar. bronchus E. Three lobes 19. Which part of the left lung might partially fill the costomediastinal recess in full inspiration? A. Apex B. Cupola C. Hilum D. Lingula E. Middle lobe 20. The oblique fissure of the right lung separates which structures? A. Lower lobe from lingula B. Lower lobe from upper lobe only C. Lower lobe from both upper and middle lobes

Page 26: Mid term exam(anatomy) 2012

系统解剖学习题(留学生) 26

D. Lower lobe from middle lobe only E. Upper from middle lobe 21. A 35­yeai­old man was stabbed in the back with a knife that just nicked his left lung halfwaybetween its apex and diaphragmatic surface. Which part of the lung was most likely injured? A. Hilum B. Inferior lobe C. Lingula D. Middle lobe E. Superior lobe 22. A 4­year­old girl is brought in with severe chest pain, and you are told by her mother that she had been playing with some beads and had apparently aspirated one (gotten it into her airway). Where would you expect it to most likely be? A. Apicoposterior segmental bronchus of left lung B. Left main bronchus C. Lingular segment of left lung D. Right main bronchus E. Terminal bronchiole of right lung, lower lobe 23. A 78­year­old female presented with edema of the left upper limb due to poor venous return. Examination revealed an aneurysm of the ascending aorta that was impinging on a large vein lying immediately anterosuperior to it, most likely the: A. Azygos v. B. Internal thoracic v. C. Left brachiocephalic v. D. Left superior intercostal v. E. Right brachiocephalic v. 24. Which statement is true about the right lung? A. It is slightly smaller than the left lung B. It has a lingular segmental bronchus C. It occupies the right most portion of the mediastinum D. Its upper lobar bronchus lies behind and above the right pulmonary artery E. It has the right phrenic nerve passing posterior to the lung root 25. During a surgical procedure in the vicinity of the descending aorta, a surgeon accidentally cuts the first aortic intercostal arteries. Which of the following structures might be deprived of its main source of blood supply? A. First posterior intercostal space B. First anterior intercostal space C. Left bronchus D. Right bronchus E. Fibrous pericardium 26. The first rib articulates with the sternum in close proximity to the: A. Nipple B. Root of the lung C. Sternal angle D. Stemoclavicular joint

Page 27: Mid term exam(anatomy) 2012

系统解剖学习题(留学生) 27

E. Xiphoid process 27. A sick person, lying supine in bed, aspirates (breathes in) some fluid into their lungs while swallowing. It would most likely end up in which of the following bronchopulmonary segments. A. Anterior segmental bronchus of right superior lobe B. Medial segmental bronchus of right middle lobe C. Superior segmental bronchus of right inferior lobe D. Medial basal segmental bronchus of left inferior lobe E. Inferior segmental bronchus of lingular lobe 28. You are caring for a 68­year­old male who has copious amounts of fluid in the left pleural cavity due to acute pleurisy. When you exam him as he sits up in bed (trunk upright) , where would the fluid tend to accumulate? A. Costodiaphragmatic recess B. Costomediastinal recess C. Cupola D. Hilar reflection E. Middle mediastinum 29. The portion the parietal pleura that extends above the first rib is called the: A. Costodiaphragmatic recess B. Costomediastinal recess C. Costocervical recess D. Cupola E. Endothoracic fascia 30. An 8­year­old boy is found to have a mid­line tumor of the thymus gland that is impinging posteriorly on a blood vessel. The affected vessel is most likely the: A. Left brachiocephalic vein B. Left pulmonary vein C. Left bronchial vein D. Right pulmonary artery E. Right superior intercostals vein 31. A frail, elderly man, suspected of having widespread cancer of the lungs and bronchi, isBrought in for bronchoscopic examination. The instrument is inserted into the airway, where itaccidentally punctures the thin, brittle posterior wall of the diseased right main bronchus. Asudden gush of blood immediately indicates that the instrument has also torn the wall of thebloodvessel immediately behind the right main bronchus, i.e. , the: A. Azygos vein B. Left brachiocephalic artery C. Pericardiacophrenic artery D. Right pulmonary vein E. Superior vena cava 32. A 10­year­old boy underwent a tonsillectomy under general anesthesia. At home he lay supine in bed for two weeks and developed a fever and chest pain with cough. He returned to thehospital and was diagnosed as having right lung pneumonia due to aspiration of infectious material during the tonsillectomy. In which bronchopulmonary segment of the lung would fluid (pus) most likely have accumulated by the simple force of gravity? A. Anterior basal segment­inferior lobe

Page 28: Mid term exam(anatomy) 2012

系统解剖学习题(留学生) 28

B. Anterior segment­superior lobe C. Lateral segment­middle lobe D. Superior segment­inferior lobe E. Superior lingual segment­lingula 33. You are observing a doctor perform a bronchoscopy. As he passes the bronchoscope down thetrachea, a cartilagenous structure is observed separating the right and left main stem bronchi.He asks what it is called? You reply that it really does look like a ship' s keel and that it iscalled, the: A. Carina B. Cricoid cartilage C. Costal cartilage D. Pulmonary ligament E. Tracheal ring 34. The minor (horizontal) fissure separates. A. The lower lobe from the lingula B. The upper lobe from the lingula C. The lower lobe from both the middle and upper lobes D. The lower lobe from the middle lobe E. The middle lobe from the upper lobe 35. A 23­year­old man comes to you complaining that he can't stop crying, i.e. tears regularlyrun down the right side of his face. You suspect that one of the lacrimal ducts on the right side ofthe face is blocked. You look into an endoscope to see if the nasolacrimal duct is blocked. Intowhich part of the nasal cavity would you look to see the opening of the duct? A. Hiatus semilunaris B. Inferior meatus C. Middle meatus D. Sphenoethmoidal recess E. Superior meatus 36. The communication between the pharynx) and the nasal cavity is known as the: A. Aditus B. Auditory tube C. Choana D. Fauces E. Piriform recess 37. The muscle most responsible for the abduction of the vocal folds is the: A, Arytenoid B. Cricothyroid C. Lateral cricoarytenoid D. Posterior cricoarytenoid E. Thyroarytenoid 38. The vocalis muscle is probably the one most responsible for the fine control of phonationbecause of its attachment into the: A. Arytenoid cartilage B. Cricoid cartilage C. Thyroid cartilage

Page 29: Mid term exam(anatomy) 2012

系统解剖学习题(留学生) 29

D. Vestibular ligament E. Vocal ligament

Answer: 1.C 2.C 3.B 4.C 5.C 6.A 7.D 8.E 9.C 10.E 11.C 12.E 13.C 14.E 15.C 16.B 17.E 18.A 19.D 20.C 21.B 22.D 23.C 24.D 25.D 26.D 27.C 28.A 29.D 30.A 31.A 32.D 33.A 34.E 35.B 36.C 37.D 38.E

Page 30: Mid term exam(anatomy) 2012

系统解剖学习题(留学生) 30

泌 mì

尿 niào

生 shēng

殖 zhí

系 xì

统 tǒng

Urinogenital system

1.A surgeon approaching the anterior aspect of the left kidney would encounter all of the following structures EXCEPT the: A. Splenorenal ligament B. Renal fascia C. Tail of the pancreas D. Left colic flexure E. Second part of the duodenum 2. The right renal artery: A. Arises from the abdominal aorta near the level of the inferior mesenteric artery B. Passes posterior to the inferior vena cava C. Supplies the only arterial branches to the right suprarenal gland D. Is shorter than the left renal artery E. Is the usual source of the right testicular (or ovarian) artery 3. The suprarenal glands: A. Are not essential for life B. Have a cortex and medulla ? C. Are drained by the inferior phrenic vein D. Are enclosed in the capsule of the kidney E. Do not have lymphatics 4. A surgeon approaching the posterior aspect of the right kidney would encounter all of the following structures EXCEPT the: A. 12th rib B. Central tendon of the diaphragm C. Quadratus lumborum muscle D. Iliohypogastric nerve E. Perirenal fat 5. A CT scan reveals a large tumor in the Superior pole of the right kidney; surgical removal of the kidney is required. What other organ ( s) are within the renal fascia? A. Spleen and tail of the pancreas B. Suprarenal gland and inferior Suprarenal artery C. 3rd part of the duodenum D. Psoas major and quadatus lumborum muscles E. None of the above is correct 6. Lymphatic vessels from the kidney and perirenal fat mainly drain: A. Along the renal vessels to lumbar lymph nodes B. Along the ureter to pelvic lymph nodes C. Toward the posterior abdominal wall D. Toward the bare area of the liver and its lymph nodes E. None of the above is correct

Page 31: Mid term exam(anatomy) 2012

系统解剖学习题(留学生) 31

7. Concerning the renal arteries: A. The right renal artery is longer than the left B. Each gives off an inferior suprarenal artery C. They are enclosed in renal fascia and surrounded with perirenal fat D. Each separates into branches which enter the renal sinus both anterior and posterior to the renal pelvis E. All of the above are correct 8. Which of the following is NOT found in the renal sinus? A. Pelvis of the ureter B. Branches of the renal vein C. Perirenal fat D. Autonomic nerve fibers E. Parietal peritoneum 9. The renal pelvis is: A. A funnel­shaped tube, wide above and' narrow below B. Formed by the junction of two or three major renal calyces C. Situated partly inside the renal sinus D. Situated partly outside the renal sinus E. All the above are true 10. Urine that leaves the distal convoluted tubule passes through the following structures in which ; Sequence? A. Collecting duct, hilus, calyces, ureter B. Collecting duct, calyces, pelvis, ureter C. Calyces, collecting duct, pelvis, ureter D. Calyces, hilus, pelvis, ureter: E. Calyces, hilus, pelvis, ureter 11. The male urethra is encircled by which structure? A. Epididymis B. Scrotum C. Prostate gland D. Seminal vesicle E. Spermatic cord 12. The urinary bladder: A. Has a base that is covered by peritoneum entirely B. Has an apex that is continuous with the puboprostatic (pubovesical) ligament C. Has lateral surfaces related to the obturator internus fascia and the levator ani muscle D. Is supplied by branches of the middle sacral artery E. In the male, has the seminal vesicles contacting its anterior surface 13. By transvaginal digital examination: A. The rectum can be palpated through the posterior vaginal wall B. The ureters can be palpated through the lateral fornices C. Pulsations of the internal iliac system of vessels can be felt through the lateral wall D. The urethra can be palpated through the anterior vaginal wall E. All of the above

Page 32: Mid term exam(anatomy) 2012

系统解剖学习题(留学生) 32

14. In the prostatic portion of the male urethra: A. Can be found the navicular fossa B. The urethral crest can be seen on the anterior wall C. The seminal colliculus can be seen on the posterior wall D. The prostatic utricle is found inferior to the seminal colliculus E. The ductus deferens enters through the lateral wall 15. In their journey from the testis to the urethra, what is the correct order in which spermatozoa Encounter the following parts of the duct system? A. Efferent ductule, rete testis, duct of epididymis, ductus deferens, ejaculatory duct B. Rete testis, efferent ductule, ductus deferens, seminal vesicle, ejaculatory duct C. Ejaculatory duct, ductus deferens, duct of epididymis, efferent ductule, rete testis D. Rete testis, efferent ductule, duct of epididymis, ductus deferens, ejaculatory duct E. Rete testis, efferent ductule, duct of epididymis, ductus deferens, seminal vesicle, ejaculatory duct 16. In its normal position, the uterus: A. Is said to be anteflexed and anteverted B. Has its intestinal surface in direct contact with the rectum C. Has coils of ileum in contact with its vesical surface D. Is covered by peritoneum on its lateral surfaces E. Has the axis of its cervical canal aligned with the axis of the vaginal canal 17. In the female, the ureters: A. Enter the superior surface of the urinary bladder B. Course along the anterior wall of the vagina C. Are in contact with peritoneum throughout their extent D. Border the posterior aspect of the ovarian fossa E. Receive their blood supply solely from the ovarian arteries 18. Concerning the urinary bladder: A. It is covered by peritoneum on all surfaces B. Its wall is traversed tangentially by the ureters C. It includes an area called the trigone in its posteroinferior wall D. All of the above are correct E. Only B and C are correct 19. The pararenal fat in the kidney bed is an elaboration of: A. Peritoneum B. Extraperitoneal connective tissue C. Transversalis fascia D. Fusion fascia E. All of above 20. All the following statements about the male genital organs are true except that: A. The testis are two glandular organs B. The epididymis consists of the head, body and tail C. The ductus deferens traverses the inguinal canal D. The ejacculatory duct open into the urinary bladder E. The prostate is around the commencement of the urethra

Page 33: Mid term exam(anatomy) 2012

系统解剖学习题(留学生) 33

21. The deep dorsal vein of the penis (clitoris): A. Passes between the arcuate pubic and transverse perineal ligaments B. Is a tributary of the internal pudendal vein C. Is a tributary of the internal iliac vein D. Is a tributary of the external pudendal vein E. Is located in the corpus spongiosum 22. The pudendal nerve: A. Arises from spinal cord levels L5, S1, S2 B. Contains no sensory nerve fibers C. Gives rise to the middle rectal nerve D. Continues as the dorsal nerve of the penis (clitoris) E. Usually gives rise to the anterior scrotal nerve in the male 23. Concerning the penis, the: A. Root (fixed portion) of the penis is attached to the perineal membrane and ischiopubic rami B. Corpus spongiosum is found dorsal to the corpora cavemosa C. Tunica albuginea is a thickened layer of connective tissue surrounding the ischiocavemosus muscles ? D. Membranous urethra contains a dilated portion called the fossa navicularis E. The glands is the expanded distal portion of the corpora cavemosa 24. A 19­year­old male suffers a tear to the psoas major muscle during the course of a football game. A scar, which formed on the medial part of the belly of the muscle, involved an adjacent nerve, immediately medial to the muscle. The nerve is called the: A. Femoral B. Genitofemoral C. Iliohypogastric D. Ilioguinal E. Obturator 25. The male pelvis tends to differ from the female pelvis in that the male pelvis often has a: A. Larger pelvic inlet B. Smaller subpubic angle C. Straighter sacral curvature D. Larger pelvic outlet E. Rounder pelvic inlet 26. After agreeing to have no more children, a man and his wife decided he should have a vasectomy. What structure would then be surgically ligated? A. Ductus deferens B. Ejaculatory duct C. Epididymis D. Fossa navicularis E. Seminal vesicle 27. Which structure does NOT form part of the boundary defining the trigone of the bladder? A. Interuteric crest B. Left ureteric orifice C. Right ureteric orifice . D. Urachus

Page 34: Mid term exam(anatomy) 2012

系统解剖学习题(留学生) 34

E. Urethral orifice 28. Which structure is found only in males? A. Anterior recess of ischoianal fossa B. Genital hiatus C. Ischiocavemosus muscle D. Rectovesical pouch E. Sphincter urethrae muscle 29. The prostate is often imaged using an ultrasound translation placed in which location? A. Penis B. Perineum C. Rectum D. Urethra E. Urinary bladder 30. The prostate gland: A. Contains upper/middle and lower lobes B. Encircles the urethra C. Is well imaged radiologically using an intravenous urogram D. Is extraperitoneal E. BandD 31. During a vasectomy, the ductus deferens is ligated in the superior part of the scrotum. Two months following this sterilization procedure, the subsequent ejaculate contains: A. Prostatic fluid only B. Seminal fluid and prostatic fluid C. Sperm only D. Sperm and seminal fluid E. Sperm, seminal fluid, and prostatic fluid 32. An elderly male patient presents with dysuria and urgency. You suspect benign prostatic hypertrophy which has caused an enlargement of the: A. Interureteric crest B. Prostatic utricle C. Seminal collicus D. Sphincter urethrae E. Uvula 33. An elderly patient notices red blood in his stool. As part of his examination, you insert a proctoscope (sigmoidoscope) through his anal canal. As you pass the scope superiorly through the rectum, the most prominent features to be seen are: A. Longitudinal muscle bands B. Tenia coli C. Transverse rectal folds D. Rectovesical pouches E. Haustra 34. An elderly patient is having difficulty in voiding (urinating). He complains that after voiding, he still feels as though he needs "to go" again. You suspect that this patient suffers from benign prostatic hypertrophy, which has caused enlargement of the __ of the bladder. A. Seminal colliculus

Page 35: Mid term exam(anatomy) 2012

系统解剖学习题(留学生) 35

B. Interureteric crest C. Ampulla D. Trigone E. Uvula 35. The part of the male reproductive tract which carries only semen within the prostate gland is the: A. Prostatic urethra B. Membranous urethra C. Seminal vesicle D. Ductus deferens E.Ejaculatory duct 36. Which of the following does not conduct spermatozoa? A. Ampulla of the ductus deferens B. Duct of the seminal vesicle C. Epididymis D. Prostatic Urethra E. All of the above 37. The suspensory ligament of the ovary; A. Connects the ovary to the uterus B. Is continuous with the round ligament C. Passes inferior to the ureter D. Contains the ovarian artery and vein E. Is firmly attached to the bifurcation of the aorta 38. The duct of the greater vestibular gland opens: A. At the posterior­lateral edge of the introitus of the vagina B. Just superior to the hymen C. At the posterior labial commissure D. Between the labium minus and labium majus E. Between the introitus and the urethra 39. Anterior extensions of the labia minima meet in the midline to form the; A. Mons pubis B. Introitus C. Body of the clitoris D. Frenulum of clitoris E Opening of the uretha 40­ You are observing a doctor perform an abdominal hysterectomy­ He notes that it is vitals protect the ureter which is found in the base of the: A. Mesometrium B. Mesovarium C. Mesosalpinx D. Round ligament of the uterus E. Suspensory ligament of the ovary 41. The extension of the vaginal lumen around the intravaginal part of the uterine cervix is the; A. Cervical canal B. Uterine lumen

Page 36: Mid term exam(anatomy) 2012

系统解剖学习题(留学生) 36

C. Fomix D. Rectouterine Pouch E. Uterovesical Pouch 42. Under normal conditions, fertilization occurs in which part of the female reproductive tract? A. Infundibulum of uterine tube B. Ampulla of uterine tube C. Isthmus of uterine tube D. Uterien lumen E. Cervical canal 43. Which structure is outlined with contrast on a CT using intraperitoneal contrast material? A. Ovary B. Prostate C. Rectum D. Seminal vesicles E. Vagina 44. Which of the following is considered a part of the broad ligament? A. Mesovarium B. Proper ovarian ligament C. Round ligament of proper uterus D. Suspensory Ligament of ovary E. Uterosacral ligament 45. A gynecologist examines a 27­year­old woman. Upon rectal examination, a firm structure, directly in front of the rectum in the midline, is palpated through the anterior wall of the rectum. This structure is the: A. Bladder B. Body of uterus C. Cervix of uterus D. Pubic symphysis E. Vagina 46. The most inferior extent of the peritoneal cavity in the female is the; A, Pararectal fossa B. Paravesical fossa C. Rectouteime pouch D. Rectovesical pouch E. Vesicouterine pouch 47. During a hysterectomy and an oophorectomy, the uterine and ovarian vessels must be"ligated. These vessels can be found in vVhich ligaments? A. Broad and proper ovarian B. Broad and suspensory C. Round and proper ovarian D. Round and suspensory E. Suspensory and proper ovarian 48. During a hysterectomy, care must be taken in ligation of the uterine vessels because they crossthe superiorly. A. Ureter

Page 37: Mid term exam(anatomy) 2012

系统解剖学习题(留学生) 37

B. Round ligament of uterus C. Ovarian artery D. Lumbosacral trunk E. Inferior hypogastric plexus 49. Following pregnancy and delivery, a 32­year­old woman continued to have problems with urinary incontinence, which developed during pregnancy. Her obstetrician counseled her to strengthen the muscle bordering the vagina and urethra, increasing its tone and exerting pressure on the urethra, This physical therapy was soon adequate to restore urinary continence. What muscle was strengthened? A. Coccygeus B. Ischiocavemosus C. Obturator Intemus D. Piriformis E. Puborectalis 50. After giving birth, a patient complains of urinary stress incontinence characterized by dribbling of urine with an increase in intra­abdominal pressure. Her physician suspects injury to the pelvic floor during delivery,' which may have altered the position of the neck of bladder and the urethra. Which muscle was most likely damaged during the vaginal delivery? A. Bulbospongiosus B. Coccygeus C. Levator ani D. Obturator intemus E. Piriformis 51. The vestibular bulbs/bulb of the corpus spongiosum are firmly attached to the; A. Inferior fascia ofurogenital diaphragm B. Sacrotuberous ligament C. Ischiopubic rami ! D. Pubic symphysis E. Ischial tuberosities 52. A structure which takes the form of a hood anterosuperior to the clitoris: A. Frenulum of clitoris B. Labia majora C. Labia minora D. Prepuce E. All of the above 53. A structure which is homologous to the male scrotum: A. Labia minora B. Labia majora C. Glans D. Shaft of corpus cavernosum E. All of the above 54. The part of the internal lining of the uterus which is shed during menstruation is the: A. Endometrium B. Myometrium C. Mesometrium

Page 38: Mid term exam(anatomy) 2012

系统解剖学习题(留学生) 38

D. Cervical mucosa E. Rugae 55. A female patient isfoundtohaveanectopic (tubal) pregnancy (embryo develops in the uterine tube). In order to gain access to the peritoneal cavity endoscopically to remove the embryo, the instrument can be passed into the vagina and through the : A. Anterior fornix B. Cervix C. posterior fornix D. Retropubic space E. Vesicouterine pouch

Answer 1.E 2.B 3.B 4.B 5.B 6.A 7.E 8.E 9.E 10.B 11.C 12.C 13.E 14.C 15.D 16.A 17.D 18.C 19.B 20.D 21.A 22.D 23.A 24.E 25.B 26.A 27.D 28.D 29.C 30.E 31.B 32.C 33.C 34.E 35.E 36.B 37.D 38.A 39.D 40.A 41.C 42.B 43.A 44.A 45.C 46.C 47.B 48.C 49.E 50.C 51.A 52.D 53.B 54.A 55.C

Viscera Define the following Terms 1) Isthmus of fauces 2) Superior alimentary canal 3) Porta hepatic 4) Hepatobiliary triangle 5) Dentate line 6) Douglas' pouch 7) Hilum of lung 8) Costodiaphragmatic recess 9) Renal hilum 10) Trigone bladder 11) Interureteric ridge 12) Spermatic cord 13) fornix of vagina Essay question 1. Describe the parts and the main structures of each parts of the pharynx and their communication. 2. Describe the shape and location of esophagus and the location of its 3 narrow parts. 3. Describe the position , the external shape and the parts of the stomach. 4. Describe the shape, the lobes and the location of liver. 5. Describe the drainage way of bile. 6. Describe the position , the shape and the lobes of the two lungs. 7. Describe the location, the shape, the structure of the two kidneys. 8.in a male patients, through which constrictions and curvatures can a small stone within the renal pelvis be excreted to the outside? 9. Describe the position , the external and internal features ,and the fixed apparatus of uterus。

Page 39: Mid term exam(anatomy) 2012

系统解剖学习题(留学生) 39

心 xīn

血 xuè

管 guǎn

系 xì

统 tǒng

Cardiovascular system

1. These are branches of abdominal aorta EXCEPT: A. The middle suprarenal arteries B. The celiac trunk C. The renal arteries D. The uterine arteries E. The ovarian arteries 2. These are structure in right atrium,EXCEPrT : A. Fossa ovalis B. Orifice of the coronary sinus C. Valve of the inferior vena cava D. Trabeculae carneae E. Crista terminalis 3. The left atrium: A. Is situated on the left side of right atrium B. Forms the left border of the heart C. Projects forwards to form the left auricle D. There are no pectinate muscle in the left atrium E. Communicates posterioinferiorly with the left ventricle 4. The blood supply of the lateral wall of left ventricle is mainly from: A. The anterior interventricular artery B. The main trunk of left coronary C. The circumflex branch D. The main trunk of right coronary E. The posterior interventricular artery 5. W hen the left atrium is enlarged which structure will be compressed first? A. Trachea B. Inferior vena cava C. Vagus nerve D. Esophagus E. Aortic arch 6. These are structures in left ventricle EXCEPT: A. Mitral valve B. Conus arteriosus C­ Trabeculae cameae D. Chordae tendineae E. Papillary muscles 7. Which of the following vein is tributary of inferior vena cave: A. The right suprarenal vein B. The left suprarenal vein C. The left ovarian vein

Page 40: Mid term exam(anatomy) 2012

系统解剖学习题(留学生) 40

D. The right ascending lumbar vein E. The portal vein 8. Which of the veins below drains the venous blood from periumbilical venous network to great saphenous vein: A Inferior epigastric vein B. Superior epigastric vein C. Thoracoepigastric vein D. Superficial circumflex vein E. Superficial epigastric vein 9. The base of heart: A. It faces forwards and to the right B. Is formed by the right atrium C. Is formed mainly by the left atrium and to a small extent, by the right atrium D. Is formed by the right ventricle E. Is formed by the left ventricle 10. The pericardium: A. Has a fibrous layer which is fused to the central tendon of the diaphragm and adventitia of the great vessels B. Has a serous layer loosely attached to the heart C. Extends superiorly to the level of the jugular notch D. Has a transverse sinus between the superior vena cava and pulmonary veins E. Has an oblique sinus between its anterior wall and the right ventricle 11. The left coronary artery: A. Courses with the middle cardiac vein B. Has an anterior interventricular branch which supplies blood to part of the interventricular septum C. Emerges from the aorta between the pulmonary trunk and the right auricle D. Has an important branch to the sinoatrial node E. Arises from the arch of the aorta 12. The coronary sinus: A. Opens into the transverse pericardial sinus B. Courses in the posterior interventricular sulcus C. Receives anterior cardiac veins as major tributaries D. Drains into the right atrium E. Is fused to the fibrous pericardium 13. A pathologist examining a heart at autopsy would find the middle cardiac vein; A. In the coronary sulcus B. In the posterior interventicular sulcus C. Draining into the left atrium D. Coursing adjacent to the left coronary artery E. Draining into the superior vena cava 14. During heart transplantation, the surgeon cuts the pericardial reflection that forms the boundary of the oblique pericardial sinus. The oblique pericardial sinus is found between the: A. Superior and inferior vena cava B. Pulmonary trunk and ascending aorta

Page 41: Mid term exam(anatomy) 2012

系统解剖学习题(留学生) 41

C. Arch of the aorta and the four pulmonary veins D. Inferior vena cava and the four pulmonary veins E. Pulmonary trunk and superior vena cava 15. The fibrous pericardium receives its major arterial supply from branches of the : A. Inferior phrenic arteries B. Coronary arteries C. Posterior intercostals arteries D.Pericardiacophrenic arteries E.Bronchial arteries 16. The right atrium : A. Is the most posterior chamber of the heart B. Has musculi pectinati on the interior of its wall C. Has a smooth vertical ridge, the terminal crest, on its septal D. Forms the apex of the heart E. Has the superior and inferior vena cava as its only sources of blood 17. Which of the following is TRUE concerning the specialized conduction system of the heart? A. Contractions of the atria are initiated at the atrioventricular node B. Stimulation of the vagus nerve tends to increase heart rate C. Activation of the ventricles is achieved by spread of the conduction impulse through the atrioventricular bundle D. The heart's fibrous skeleton forms a complete electrical barrier between the atria and ventricles E. The septomarginal trabecula carries the conduction impulse across the membranous portion of lhe interventricular septum 18. A deficiency in blood flow to the heart's sinuatrial node would probably result from blocking blood flow through which of the following? A. Anterior interventricular artery B. Circumflex artery C. Right coronary artery D. Right marginal artery E. Posterior interventricular artery 19. The interventricular septum : A. Has bolh muscular and membranous parts B. Receives its only arterial supply from the posterior interventricular artery C. Is concave on the right side when viewed in cross section D. Does not have a papillary muscle attached to it E. Is lined with visceral pericardium 20. A pediatric surgeon repairing a patent (open) forameen oval must be aware that the sinoatrialnode is located : A. Near the orifice of the mitral valve B. In the crista termnalis near the opening of the superior vena cava C. In the interatrial septum D. ln the moderator band E. Between the opening of the inferior vena cava and the coronary sinus 2l. Thepectinatemuscles arelocated within the heart:

Page 42: Mid term exam(anatomy) 2012

系统解剖学习题(留学生) 42

A. On the anterior wall of the atrium B. On the interatrial septum C. In the right ventricle D. On the sinus venarum E. On the right side of the interventricular septum 22. The aortic semilunar valve : A. Usually has four cusps B. Is located anterior to the pulmonary C. When closed, prevents backflow of blood into the left ventricle D. Is superior to the origin of the coronary arteries E. Is supported by chordae tendineae 23. The apex of the heart is usually located deep to the : A. Xiphisternum B. 5th intercostals space about 8cm left of the midline C. Junction of the 5th costal cartilage with the sternum D. Right 4th intercostals space E. Left 4th intercostals space adjacent to the sternum 24. The thoracic aorta : A. Is the origin of all intercostals arteries B. Enters the abdomen behind the median arcuate ligament at the level of the 12lh thoracic ventebra where it becomes the abdominal aorta C. Courses anterior to the esophagus D. Courses to the right of the midline E. Has only paired branches in the posterior mediastinum 25. Which of the following is FALSE concerning the azygos system of veins? A. The azygos vein collects blood from right intercostals veins B. In the thorax, the azygos vein usually ascends on the vertebral column to the right of the midline C. The thoracic duct is between the aorta and the azygos vein D. The hemiazygos vein is formed by the junction of the left ascending lumbar and subcostal veins E. The azygos vein is crossed anteriorly by the right posterior intercostal veins 26. The brachiocephalic veins are situated in the : A. Superior mediastinum B. Anterior mediastinum C. Middle mediastinum D. Posterior mediastinum E. Retropharyngeal mediastinum 27. In coarctation of the aorta the ribs may be notched due to enlarged collateral blood vessels. These collateral anastomoses would involve all of the following vessels EXCEPT the : A. Subclavian arteries B. Highest intercostals arteries C. Internal thoracic arteries D. Inferior thyroid arteries E. Anterior intercostal arteries

Page 43: Mid term exam(anatomy) 2012

系统解剖学习题(留学生) 43

28. Which of the following does NOT drain into a brachiocephalic vein? A. Inferior thyroid vein B. Vertebral vein C. Arch of the azygos vein D. Left superior intercostals vein E. Right internal thoracic vein 29. In reference to the lymphatics of the abdomen: A. The thoracic duct receives lymph from the lower extremities only B. Very few lymph nodes are associated with the abdominal aorta C. Lymph from the intestine may be whitish after a fatty meal due to absorption of fat D. Lymphatic return from the GI tract is limited because the GI tract has a sparse lymphatic capillary bed E. The cistema chyli is located at the level of the 4th lumbar vertebra 30. On its course to the cistema chyli, lymph from the lower extremity could pass through which of the following lymph nodes? A. External iliac nodes B. Common iliac nodes C. Lumbar nodes D. Inguinal nodes E. All of the above 31. The ovarian artery: A. Arises as a branch of the abdominal aorta on both the left and right sides B. Courses through the mesometrium C. Supplies blood to the anal canal D. Has an anastomosis with the inferior epigastric artery E. Passes inferior to the ureter to on its way to the uterus 32. Blockage of which of the following arteries would lead to ischemia of the apex of the heart? A. Anterior interventricular (descending) B. Left circumflex C. Posterior interventricular (descending) D. Right marginal E. Right coronary 33. If the ductus arteriosus does not spontaneously close off soon after birth (to become the ligamentum arteriosum), it may have to be surgically ligated. When clamping or ligating it, whatimportant structure immediately behind it must be identified and saved? A. Arch of the azygos vein B. Internal thoracic artery C. Left phrenic nerve D. Left recurrent laryngeal nerve E. Left superior intercostal vein 34.A hand slipped behind the heart at its apex can be extended upwards until stopped by a line of pericardial reflection that forms the : A. Cardiac notch B. Costomediastinal recess C. Hilar reflection

Page 44: Mid term exam(anatomy) 2012

系统解剖学习题(留学生) 44

D. Oblique pericardial sinus E. Transverse pericardial sinus 35. Which valves would be open during ventricular systole? A. Aortic and pulmonary B. Aortic and tricuspid C. Mitral and aortic D. Tricuspid and mitral E. Tricuspid and pulmonary 36. Which statement is true of the right atrioventricular valve? A. It is also called the mitral valve B. It is open during ventricular diastole C. It transmits oxygenated blood D. It is opened by the pull of chordae tendineae E. It consists of 2 leaflets 37. An elderly lady suffers a coronary occlusion and subsequently it is noted that there is a complete heart block (that is, the right and left bundles of the conduction system have been damaged. The artery most likely involved is the: A. Acute marginal branch B. Circumflex branch C. Anterior interventricular (Left anterior descending) D. Obtuse marginal E. Posterior interventricular (posterior descending) 38. Which event occurs during ventricular diastole? A. Filling of coronary arteries B. Closure of mitral valve C. Opening of pulmonary valve D. Closure of tricuspid valve E. Opening of aortic valve 39. The sound associated with tricuspid stenosis (narrowing) in a 40­year­old male would be best heard at which location on the anterior chest wall? A. Below the left nipple B. In the right 2nd intercostal space near the sternum C. Over the apex of the heart D. Over the sternal angle E. Xiphoid area, just off the sternum 40. Blockage of blood flow in the proximal part of the anterior interventricular artery could deprive a large area of heart tissue of blood supply, unless a substantial retrograde flow into this artery develops via an important anastomosis with which other artery? A. Circumflex B. Left marginal C. Posterior interventricular D. Right coronary E. Right marginal 41. Traumatic, acceleration/deceleration injuries to the aorta usally occur where its mobile and fixed portions meet. This would be at the:

Page 45: Mid term exam(anatomy) 2012

系统解剖学习题(留学生) 45

A. At the ligamentum arterious B. Junction of aortic arch with the descending portion C. Junction of the ascending aorta with the heart D. Origin of the brachiocephalic artery on the arch E. Point where the descending aorta passes through the diaphragm 42. Which structure does NOT lie in the coronary sulcus? A. Circumflex artery B. Coronary sinus C. Right coronary artery D. Right marginal artery E. Leit coronary artery 43. During a hysterectomy and an oophorectomy, the uterine and ovarian vessels must be ligated. These vessels can be found in which ligaments? A. Broad and ovarian B. Broad and suspensory C. Round and ovarian D. Round and suspensory E. Suspensory and ovarian 44. During a hysterectomy, care must be taken in ligation of the uterine vessels because they cross the __ superiorly. A. Ureter B. Round ligament of the uterus C. Ovarian artery D. Lumbosacral trunk E. Inferior hypogastric plexus 45. Which of the following nerves would be most vulnerable to irritation when the tracheobronchial lymph nodes are enlarged due to a diseased situation? A. Right phrenic B. Left phrenic C. Right recurrent laryngeal D. Left recurrent laryngeal E. Right vagus 46. In obstruction of the superior or inferior vena cava, venous blood is returned to the heart by an alternate route via the azygos vein, which becomes dilated in the process. Which of the following structures might it compress as a result? A. Trachea B. Root of the left lung C. Phrenic nerve D. Thoracic duct E. Descending aorta 47. While attempting to suture the distal end of a coronary bypass onto the anterior mterventricu­ lar artery, the surgeon accidentally passed the needle through the adjacent vein. Which vein was damaged? A. Anterior cardiac vein B. Coronary sinus

Page 46: Mid term exam(anatomy) 2012

系统解剖学习题(留学生) 46

C. Great cardiac vein D. Middle cardiac vein E. Small cardiac vein 48. The heart sound associated with the mitral valve is best heard: A. In the jugular notch B. In the second left intercostal space C. In the second right intercostal space D. In the fifth left intercostal space E. To the right of the xyphoid process 49. Lymphatic drainage of the terminal portion of the gastrointestinal tract may flow initially intoeither the superficial inguinal nodes or the pararectal nodes, depending upon whether thelymph is formed above or below the: A. Anorectal Junction B. Muscular Sling of the Puborectalis Muscle C. Pectinate Line D. White Line E. Cutaneus Zone 50. A 64­year­old woman was diagnosed as having carcinoma of the distal gastrointestinal tract. At surgery, lymph nodes from the sacral, internal iliac and inguinal lymph node groups were removed and sent to pathology for study. Only the superificial inguinal nodes contained cancerouscells. In which part of the GI tract was the tumor localized? A. Cutaneous portion of anal canal B. Distal rectum C. Mucosal zone of anal canal D. Pectinate line of anal canal E. Proximal rectum 51. A malignant tumor in the cutaneous zone of the anal canal would most likely metastasize (spread) to which group of lymph nodes? A. Inferior mesenteric B. Pararectal C. Sacral D. Superficial inguinal E. Superior mesenteric 52. During a hysterectomy, the uterine vessels are ligated. However­ the patient's uterus continues to bleed. The most likely source of blood still supplying the uterus is from which artery? A. Inferior vesical B. Internal pudendal C. Middle rectal D. Ovarian E. Superior vesical 53. Blood supply to the superior portions of the bladder typically arises from the __ arteries. A. Umbilical B. Middle rectal C. Obturator

Page 47: Mid term exam(anatomy) 2012

系统解剖学习题(留学生) 47

D. Inferior gluteal E. Uterine 54. An intrahepatic blockage of the portal venous outflow may cause intestinal blood to­ drain via portal­systemic anastomoses into the: A. Superior gluteal vein B. Inferior rectal vein C. Splenic vein D. Renal vein E. Inferior phrenic vein 55. If the venous drainage of the anal canal above the pectinate line is impaired in a patient with portal hypertension, there may be an increase in blood flow downward to the systemic venous system via anastomoses with the inferior rectal vein, which is a tributary of the: A. External iliac B. Inferior gluteal C. Inferior mesenteric D. Internal iliac E. Internal pudendal 56. A patient complains of a boil located on her labia majora. Lymphatic spread of the infection' would most likely enlarge which nodes? A. Lumbar nodes B. Sacral nodes C. External iliac nodes D. Superficial inguinal nodes E. Internal iliac nodes 57. Most of the drainage of the thoracic body wall reaches the superior vena cava via the azygos vein. A notable exception is the left superior intercostal vein, which normally drains into the: A. Left brachiocephalic vein B. Left bronchial vein C. Left pulmonary vein D. Left subclavian vein E. Superior vena cava

Answer 1.D 2.D 3.C 4.C 5.D 6.B 7.A 8.E 9.C 10.A 11.B 12.D 13.B 14.D 15.D 16.B 17.C 18.C 19.A 20.B 21.A 22.C 23.B 24.B 25.E 26.A 27.D 28.C 29.C 30.E 31A 32.A 33.D 34.D 35.A 36.B 37.C 38.A 39.E 40.C 41.A 42.D 43.A 44.A 45.D 46.D 47.C 48.D 49. C 50.A 51.D 52.D 53.A 54.B 55.E 56.D 57.A

Angiology

Define the following Terms 1) systemic circulation 2) pulmonary circulation

Page 48: Mid term exam(anatomy) 2012

系统解剖学习题(留学生) 48

3) Fossa ovalis 4) Septomarginal trabecula 5) conduction system 6) pericardial cavity 7) arterial ligament 8) Carotid sinus 9) Carotid glomus 10) Danger triangle of the face 11) Venous angle 12)Thoracic duct

Essay question 1) Describe the position and the shape of the heart. 2) Describe the structures of cardiac cavity. 3) Describe the composition, structure, position and function of conduction system of heart. 4) Describethe structural characteristics of the hepatic portal vein and the anastomoses between the hepatic portal vein and vena cava. 5) if the medicine is injected into the cephalic vein , how does it be conveyed to the appendix? 6) how does the medicine injected into the saphenous vein get to the lungs? 7) after oral administration of berberine, the urine of patients became yellow .how were the yellow metabolic products excreted to the outside of body?

Page 49: Mid term exam(anatomy) 2012

系统解剖学习题(留学生) 49

感 gǎn

觉 jiào

系 xì

统 tǒng

Sensing system

l. When examining the eye of your patient, the : A. Lacrimal gland can be seen draining into the inferior coqunctival fonix B. Blood vessels of the sclera can be seen through the bulbar conunctiva C. Lacrimal papiIlae are normally not visible D. Lacrimal sac can be seen and palpated E. None of the above 2. Concerning the orbit : A. The lateral wall is parallel to the medial wall B. The inferior orbital fissure Communicates directly between the orbit and the middle cranial fossa C. The medial wall is thin and prone to “blow out ” fractures D. The anterior ethmoidal foramen transmits branches of the maxillary division of the trigeminal nerve (V2) E. The optic canal communicates directly between the orbit and the anterior cranal fossa 3. When coursing from the middle cranial fossa to the infraorbital region, the maxillary division of the trigeminal nerve: A. Passes through the foramen rotundum B. Courses into the pterygopalatine fossa C. Gives rise to pterygopalatine branches which go the pterygopalatine ganglion D. Sends a branch into the orbit through the infraorbital fissure E. All of the above 4. The infraorbital foramen : A. Pierces the zygomatic bone B. Transmits a nerve that is a branch of the maxillary division of the trigeminal nerve (V2) C. Is located directly posterior to the zygomaticus major muscle D. Transmits an artery that is a branch of the ophthalmic artery E. None of the above 5. Which of the following nerves supplies the orbicularis oculi muscle? A. Zygomatic branch of the facial nerve B. Temporal branch of the facial nerve C. Buccal branch of the facial nerve D. Infraorbital branch of the trigeminal nerve E. Both A and B supply branches to this muscle 6. The lateral wall of the orbit: A. Is parallel to the median sagittal plane B. Is formed in part by the maxilla C. Is the thinnest wall of the orbit and is therefore susceptible to "blow out" fractures D. Is formed in part by the zygomatic bone E. Is the site of the superior orbital fissure

Page 50: Mid term exam(anatomy) 2012

系统解剖学习题(留学生) 50

7. Concerning the lacrimal apparatus: A. The lacrimal gland is located superficial to the orbital septum. B. The lacrimal ducts open at the superior conjunctival fomix. C. The lacrimal canaliculi open directly into the inferior meatus of the nasal cavity. D. Sensory fibers that supply the lacrimal gland travel in the maxillary division of the trigeminal nerve (V2) E. The major blood supply to the lacrimal gland is via the angular artery 8. The oculomotor nerve: A. Courses medial to the internal carotid artery in the cavernous sinus B. Contains preganglionic parasympathetic fibers that supply the lacrimal gland C. Supplies the motor innervation to the inferior oblique muscle D. Has a branch that travels through the anterior ethmoidal foramen E. Enters the orbit superior to the common annular tendon 9. The superior oblique muscle. A. Takes origin from the common annular tendon B. Inserts into the superior posterior lateral surface of the eyeball C. Is located superior to the frontal nerve D. Is supplied by the oculomotor nerve E. Runs paraIlel to the lateral wall of the orbit 10. The ciliary ganglion: A. Is normally located medial to the optic nerve B. Is the site of sympathetic nervous system synapses C. Receives preganglionic parasympathetic fibers that travel in the oculomotor nerve (Ⅲ) D. Receives preganglionic parasympathetic fibers that travel in the facial nerve (Ⅶ) E. Is the site of synapses that control secretion of the lacrimal gland 11. Concerning the ear: A. Pain from the tympanic membrane are carried by cranial nerve Ⅶ B. The tensor tympani and stapedius muscles function to amplify sounds C. The handle of the malleus covers the fenestra cochleae (round window) D. The posterior cranial fossa lies superior to the tegmen tympani E. The promontory is actually the basal turn of the cochlea 12. All of the following are features of the medial wall of the tympanic cavity EXCEPT the: A. Fenestra vestibuli B. Prominence of the lateral semicircular canal C. Prominence of the facial nerve D. Pyramidal eminence E. Tympanic plexus 13. The internal acoustic meatus houses which cranial nerves? A. Facial nerve B. Trigeminal nerve C. Vestibulocochlear nerve D. A and B E. A and C 14.The oval window is the site of attachment of which middle ear bone ? A. Malleus

Page 51: Mid term exam(anatomy) 2012

系统解剖学习题(留学生) 51

B. Arytenoid C. Cricoid D. Stapes E. Incus 15.Which of the following is NOTT involved in sensory innervation of the tympanic membrane? A. Trigeminal ganglion B. Geniculate ganglion C. Vestibulocochlear nerve D. Glossopharyngeal nerve E. Vagus nerve 16. When you look into the external acoustic meatus to examine the tympanicmembrane, youcan see: A. The umbo B. Skin that is covering both bone and cartilage C. The cone of light D. Skin that is innervated by the facial nerve E. All of lhe above 17. The spiral organ is situated in: A. The saccule B. The utricle C. The posterior semicircular duct D. The cochlear duct E. The anterior membranous ampulla 18. The entry of bacteria through which space could lead to an infection in the mastoid air cells. A. Auditory (nasopharyngeal) tube B. Cochlea C. External acoustic meatus D. Internal acoustic meatus E. Sacculus 19. Which structure is attached to the center of the tympanic membrane? A. Foot plate of the stapes B. Handle (manubrium) of the malleus C. Long process of the incus D. Tragus E. Utricle 20. A 45­year­old woman with recurrent left middle ear infection (otitis media) complained of partial dryness of her mouth to her ENT surgeon. Taste sensation and hearing were normal. After a thorough clinical examination at the hospital; the doctor concluded that the infection must have spread to a component of the glossopharyngeal nerve (CN E ) that supplies the parotid gland. On which of the following walls of the middle ear is this nerve component located? A. Anterior B. Lateral C. Medial D. Posterior E. Roof

Page 52: Mid term exam(anatomy) 2012

系统解剖学习题(留学生) 52

21. A 3­year­old girl ruptured her eardrum when she inserted a pencil into her ear. Her mother took her to the emergency department after noticing that the child was crying and complaining of pain in her ear with a few drops of blood in the external auditory meatus. The attending doctor examined the child for possible injury to a nerve that runs across the eardrum. The most likely nerve to be injured is the A. Auricular branch of the vagus B. Chorda tympani C. Glossopharyngeal (CN Ⅸ) D. Lesser petrosal E. Trigeminal (CN V) 22. An elderly patient with chronic otitis media (middle ear infection) might have all the follwing complications EXCEPT: A. Inability to chew food due to injury to the mandibular division of the trigeminal nerve(CN V) B. Loss of taste in the anterior part of the tongue due to injury to the chorda tympani nerve C. Mastoiditis D. Paralysis of facial muscles due an injury to the facial nerve (CN Ⅶ) E. Some degree of deafness due to damage to the ossicles 23. A patient with a facial nerve paralysis suffers from inability to dampen loud noises (hyperacu­ sis) due to denervation of which muscle? A. Posterior belly of digastrics B. Stampedes C. Tensor tympani D. Stylohyoid muscle E. Digastric 24. All of the following are true about the middle ear EXCEFT: A. The joints between ossicles are synovial B. The chorda tympani nerve is related to the lateral wall C. The facial nerve passes in a canal situated in the medial and anterior walls D. The auditory tube connects the nasopharynx with the anterior wall E. Its mucous membrane is supplied by the glossopharyngeal nerve (CN E) 25. A patient has sustained a fracture to the base of the skull. Thorough examination concluded that the right greater petrosal nerve, among other structures; has been injured. This conclusion was based on which of the patient's signs: A. Partial dryness of the mouth due to lack of salivary secretions from the submandibular and sublingual glands. B. Partial dryness of­the mouth due to lack of salivary secretions from the parotid gland. C. Dryness of the right cornea due to lack of lacrimal secretion. D. Loss of taste sensation from the right anterior 2/3rd of the tongue. E. Loss of general sensation from the right anterior 2/3rd of the tongue. 26.The location of the otic ganglion is in the : A. Pterygopalatine fossa B. Internal ear C. Infratemporal fossa D. Middle cranial fossa E. None of the above

Page 53: Mid term exam(anatomy) 2012

系统解剖学习题(留学生) 53

27. A patient complains of loss of hearing in the right ear.. examination reveals ankylosis (otoscle­rosis) of the footplate of the stapes to the surrounding bone. Which part of the bony labyrinth is involved? A. Aditus ad antrum B. Cochlear (round) window C. Cochlear duct D. Internal acoustic meatus E. Vestibular (oval) window 28. Repeated middle ear infections have destroyed the tympanic plexus in the middle ear cavity. The loss of preganglionic parasympathetic fibers that pass through the plexus diminish production of: A. Mucus in the nasal cavity B. Mucus on the soft palate C. Saliva by the parotid gland D. Saliva by the submandibular and sublingual glands E. Tears by the lacrimal gland 29. You are testing the extraocular muscles and their innervation in a patient who periodicallyexperiences double vision. When you ask him to turn his right eye inward toward his nose andlook downward he is able to look inward, but not down. Which nerve is most likely involved? A. Abducens B. Nasociliary C. Oculomotor, inferior division D. Oculomotor, superior division E. Trochlear 30. The outermost layer of the optic nerve sheath is a continuation of the: A. Arachnoid membrane B. Meningeal dura C. Periosteal dura D. Pia mater E. Retina 31. The inner lining of the eyelid is called the : A. Orbital septum B. Palpebral conjunctiva C. Periorbita D. Sclera E. Tarsal plate 32. What would the examining physician notice in the eye of a person who has taken a sympathetic blocking agent? A. Exophthalmos and dilated iris B. Enophthalmos and dry eye C. Dry eye and inability to accommodate for reading D. Wide open eyelids and loss of depth perception E. Ptosis and miosis (pin­point iris)

Page 54: Mid term exam(anatomy) 2012

系统解剖学习题(留学生) 54

33. You are examining a patient who has a pituitary tumor involving the cavernous sinus. While doing a preliminary eye exam, you suspect the right abducens nerve of the patient has been damaged by the tumor. In which direction would you have the patient turn his right eye to confirm the defect? A. Inward B. Outward C. Downward D. Down and out E. upward 34. You have a patient with a droping right eyelid. You suspect Homer's syndrome. Which of the following signs on the right side would confirm a diagnosis? A. Constricted pupil B. Dry eye (Lack of tears) C. Exophthalmos D. Pale, blanched face E. Sweaty face 35. Following endarterectomy on the right common carotid, a patient is found to be blind in the right eye. It is appears that a small thrombus embolism during surgery and lodged in the artery supplying the optic nerve. What artery would be blocked? A. Central artery of the retina B. Infraorbital C. Lacrimal D. Nasociliary E. Supraorbital 36.You are asked to check the integrity of the trochlear nerve in the right eye of a patient, Starting with the eyes directed straight ahead,you would have the patient look: A. Inward, toward the nose and downward B. Inward, toward the nose and upward C. Toward the nose in a horizontal plane D. Laterally in a horizontal plane E. Outward,away from the nose and downward 37. The ducts of the lacrirnal gland open into the: A. Superior fornix of the conjunctiva B. Inferior fornix of the conjunctiva C. Lacrimal puncta D. Lacrimal canaliculi E. Lacrimal lake 38. Starting from a position gazing straight ahead,to direct the gaze downward, the inferior rectus muscle must be active along with the: A. Superior oblique B. Inferior oblique C. Medial rectus D. Lateral rectus A. Superior rectus

Page 55: Mid term exam(anatomy) 2012

系统解剖学习题(留学生) 55

39. During a physical examination it is noted that a patient has ptosis. What muscle must be paralyzed? A. Orbicularis oculi?lacrimal part B. Orbicularis oculi, palpebral part C. Stapedius D. Superior oblique E. Superior tarsal 40. An adolescent boy suffers from severe acne.As is often the case he frequennlly squeezed the pimples on his face. He subsequently develops a fever and deteriorates into a confused mental state and drowsiness. He is taken to his physician and after several tests a diagnosis of cavernous sinus infection and thrombosis is made. The route of entry to the cavernous sinus from the face was most likely the: A. Carotid artery B. Mastoid emissary vein C. Middle meningeal artery D. Ophthalmic vein E. Parietal emissary vein

Answer

1.B 2.C 3.E 4.B 5.E 6.D 7.B 8.C 9.B 10.C 11.E 12.D 13.E 14.D 15.C 16.E 17.D 18.A 19.B 20.C 21.B 22.A 23.B 24.C 25.C 26.C 27.E 28.C 29.E 30.B 31.B 32.E 33.B 34.A 35.A 36.A 37.A 38.A 39.E 40.D

The sensory organs

Define the following Terms 1) Optic disc 2) Macula lutea 3) spinal organ

Essay question 1) Describe the wall of eyeball and it’s parts。 2) Describe the circulation of aqueous humor. 3) Describe the composition of vestibulocochlear organ 4) Describe the auditory pathways in normal situation.

Page 56: Mid term exam(anatomy) 2012

系统解剖学习题(留学生) 56

中 zhōng

枢 shū

神 shén

经 jīng

系 xì

统 tǒng

Central nervous system

1. A dermatome: A. Is a term describing the thickness of the dermis B. Refers to the region of distribution of a named cutaneous nerve C. Refers to an area of skin supplied by nerve fibers arising within a single dorsal root ganglion D. Is a specialized area of skin occurring only on the trunk of the body E. None of the above 2. Regarding spinal nerves: A. Once formed, they usually begin branching while still within the vertebral canal B. Each is very different so that there is no typical pattern C. The eighth pair of thoracic nerves exits between thoracic vertebrae T8 and T9 D. The first pair (cervical 1) exits between the first and second cervical vertebrae E. All spinal nerve roots are equal in length 3. Which of the following structures, if lesioned, ould cause the greatest functional deficit? A. A dorsal root B. A ventral root C. A spinal nerve D. A spinal ganglion (dorsal root ganglion) E. An anterior primary ramus 4. Ganglia: A. Are found only on dorsal roots of the spinal nerves T1­T12 B. That are sensory in function are usually located outside of the vertebral canal C. Are swellings on dorsal primary rami D. Are sites of synapses of axons carrying impulses to the central nervous system E. None of the above 5. Which statement is TRUE about the deep back muscles? A. The erector spinae flexes (bends anteriorly) the vertebral column B. They are innervated by many levels of the spinal cord through dorsal primary ami C. It is possible to perform a laminectomy without disturbing the deep back muscles D. Iliocostalis is the most medial of the three columns of erector spinae muscles E. The erector spinae muscles lie anterior to the vertebral transverse processes 6. With regard to the vertebral column: A. The sacrum is formed from three fused vertebrae B. The spinous process of a typical thoracic vertebra tends to be square in shape C. The spinous process of vertebra Tr is called the vertebra prominens D. The supraspinous ligament courses from one adjacent lamina to another E. Lordosis is an abnormal condition in which there is an increase in the posterior concavity of the lumbar vertebral column 7. Spinal dura mater: A. Is the periosteum of the vertebral canal B. Is tightly fused to the arachnoid

Page 57: Mid term exam(anatomy) 2012

系统解剖学习题(留学生) 57

C. Extends along the spinal nerves as the endoneurium D. Encloses a sac which extends inferiorly to the level of the second sacral vertebra E. Is a thin,delicate structure 8. Included among the structures within the vertebral canal at the appropriate level for a spinal tap are all of the following EXCEPT: A. Nerve roots B. Filum terminale C. Denliculate ligaments D. Veins of the vertebral venous plexus E. All of the above 9. Ligaments that a needle would normally pass through in reaching the vertebral canal include all of the following EXCEPT: A. Interspinous B. Ligamentum flavum C. Anterior longitudinal D. Supraspinous E. All of the above 10. A patient has a virus that has infected the ventral horns of the spinal cord from T6 to the conus medullaris,damaging all of the neurons that reside there. This condition would result in all of the following conditions EXCEPT: A. Paralysis of the posterior inferior serratus muscle B. Inability to rotate the lumbar vertebrae relative to the pelvis C. Paralysis of the lower portion of the trapezius muscle D. In time, a reduction in the depth of the vertebral furrow E. Loss of ability to extend the lumbar spine 11. If the anterior root of a spinal nerve were cut, what would be the result in the tissue or region that nerve supplies? A. Complete loss of movement B. Complete loss of sensation C. Complete loss of sensation and movement D. Loss of neither sensation nor movement, but only of autonomic control of bloodvessels and sweat glands E. None of the above l2. A collection of nerve cell bodies outside the central nervous system is refered to as a: A. Ganglion B. Horn C. Nucleus D. White matter E. Gray matter 13. Neurons that conduct impulses from the central nervous system to muscles or glands are referred to as: A. Afferent B. Association C. Voluntary D. Efferent

Page 58: Mid term exam(anatomy) 2012

系统解剖学习题(留学生) 58

E Sensory 14. A neuron that transmits a nerve impulse to the central nervous system is called a/an: A. Motor B. Sensory C. Bipolar D. Association E. Multipolar 15. The hypothalamus is part of the: A. Basal nuclei B. Diencephalon C. Cerebrum D. Cerebellum E. Teleceptor 16. Which of the following combinations is NOT correct? A. Corpus callosum­diencephlon B. Corpora quadrigemina­midbrain C. Cortex­cerebellum D. Thalamus­diencephalon E. Vermis­cerebellum 17. Which of the statements on the motor neurons in spinal cord is correct: A. They are smaller neurons in anterior horn B. They supply the muscle fibers of neuromuscular spindles C. They are located at posterior horn D. They are called the upper motor neurons E. Their efferent fibers emerge via the ventral root 18. The level of 10th thoracic segment of spinal cord corresponds with. A. The body of 5th thoracic vertebra B. The body of 6th thoracic vertebra C. The body of 7th­ thoracic vertebra D. The body of 8th thoracic vertebra E. The body of 10th thoracic vertebra 19. Rami communicantes: A­ Are branches of anterior primary rami of thoracic but not of cervical nerves B. Designated by the term "white" are composed of axons of postganglionic neurons C. Formed by axons of preganglionic neurons are branches of ventral primary rami of thoracic spinal nerves D. Branch directly from ventral spinal nerve roots E. Are involved with innervation of striated or skeletal muscle 20. A football player suffers a hemiated (ruptured) intervertebral disk in his neck. The disk compresses the spinal nerve exiting through the intervertebral foramen between the 5th and 6th cervical vertebrae. Which spinal nerve is affected? A. C4 B. C5 C. C6 D. C7

Page 59: Mid term exam(anatomy) 2012

系统解剖学习题(留学生) 59

E. C8 21. A man has a hemiated intervertebral disk between the fourth and fifth lumbar vertebrae. If this disk compresses the spinal nerve in the intervertebral foramen immediately posterior to this disk, which spinal nerve would be affected? A. L3 B. L4 C. L5 D. S1 E. S2 22. Kyphosis is an accentuated or abnormal curvature of whjch region of the spine? A. Cervicle B. Thoracic C. Lumbar D. Sacral E. Coccygeal 23.Both the dural sac and the subarachnoid space end at which vertebral level? A. L4 B. L5 C. S2 D. S1 E. S4 24.The number of vertebrae and num ber of spinal cord segments are the same in each region EXCEPT: A. Cervical B. Thoracic C. Lumbar D. Sacral E. All of the above 25.The spinal cord issegmented like the vertebral column,but it contrast to the vetebrae, there are only Cord segments. A. 28 B. 29 C. 30 D. 31 E. 32 26. In the lumbar spine,the L4 nerve root stleeve exits: A. Above the pedicle of L4 and at the top of the intervertebral foramen B. Above the pedicle of L4 and at the bottom of the intervertebral foramen C. Below the pedicle of L4 and at the top of the intervertebral foramen D. Below the pedicle of L4 and at the bottom of the intervertebral foramen E. All of above 27. Which structure does NOT contain efferent autonomic nerve fibers? A.Dorsal ramus of C4 B.Dorsal root of T6 C.Ventral root of T3

Page 60: Mid term exam(anatomy) 2012

系统解剖学习题(留学生) 60

D.Ventral ramus of L2 E.Dorsal ramus of C8 28. In an adult, the conus medullaris of the spinal cord is normally positioned at which vertebral body levels: A. Pineal body B. Corpus callosum C. Lateral ventricle D. Basal nuclei E. Hippocampus formation 29. which of the following is NOT part of the cerebrum? A.Pineal body B.Corpus callosum C.Lateral ventricle D.Basal nuclei E.Hippocampus formation 30. A neuron that transmits an impulse away from the CNS is caIled a/an neuron: A. Association B. Motor C. Sensory D. Afferent E. Efferent 31. The central sulcus separates the : A. Frontal lobe from the parietal B. Parietal lobe from the occipital lobe C. Parietal lobe from the temporal lobe D. Occipital lobe from the cerebullum E. Frontal lobe from the temporal lobe 32. Regarding the cranial nerves: A.The optic nerve is accompanied by the ophthalmic artery through the optic canal B. The cranial part of the accessory nerve is sensory to the mucosa of the pharynx C. The sigmoid venous sinus becomes the internal jugular vein at the jugular foramen D. The oculomotor nerve transmits parasympathetic fibers to the lacrimal gland E. The facial nerve transmits taste fibers from the posterior third of the tongue 33.How many lobes can the hemisphere of telencephalon be divided by the sulcus and an imaginary line? A. 3 B. 6 C. 5 D. 2 E. None of the above 34. Which of the following about limbic system is wrong? A. Emotional expression and genesis,together with visceral response to emotions B. Survival of individual and species C. Cognitive processes with involved in memory D. Visceral brain

Page 61: Mid term exam(anatomy) 2012

系统解剖学习题(留学生) 61

E. None of the above 35.Where is the somesthetic area? A. Area 3, 1 and 2 of Brodmann’s map B. Area 4 and 6 of Brodmann’s map C. Area 41 and 42 of Brodmann’s map D. Area 16 and 17 of Brodmann’s map E . Area 9, 10 and 12 of Brodmann’s map 36.Where is the motor area? A. The anterior wall of the central sulcus and the anterior of the paracentral lobule on the medial surface of the hemisphere B. The posterior wall of the central sulcus and the poslerior of the paracentral lobule on the medial surface of the hemisphere C. The superolateral surface and the posterior of the paracentral lobule on the medial surface of the hemisphere D. The superolateral surface and the posterior of the anterior lobule on the medial surface of the hemisphere E. Medial surface of the occipital lobe 37.Which of the fol1owing did not belong to the language areas? A. The sensory 1anguage area B. The visual language area C. The motor language area D. The writing language area E. None of the above 38. Which of the following is the basal nuclei? A. Corpus striatum B. Subthalamus C. The reticular nucleus D. Habenular nuclei E. Lateral geniculate nucleus 39. Which of the following about the corpus callosum is TRUE: A. It can be divided into 4 parts: the rostrum, genu, body and splenium B. Belongs to the association fibers C. Belongs to the projection fibers D. Connect and transmit nerve impulses between gyri in the same hemisphere E. The corpus capsule is connections between the cortex and the subcortical structures 40. Which of the following about internal capsule is wrong? A. Internal capsule is connections between the cortex and the subcortical structures B. Internal capsule is a concentrated structure of the projection fibers C. Consists of an anterior limb, a genu, and a posterior limb D. Belongs, to the projection fibers E. None of the above 41. Which of the following about the lateral ventricles is wrong? A. They are roughly C­shaped cavities lined by ependymal epithelium B. They are in each cerebral hemisphere C. They filled with cerebrospinal fluid

Page 62: Mid term exam(anatomy) 2012

系统解剖学习题(留学生) 62

D. Each lateral ventricle consists of a body in the region of the parietal lobe from which anterior, posterior and inferior horn E. None of the above 42. The visual area: A. Surrounds the calcarine sulcus on the medial surface of the frontal lobe B. The visual area is in area 41 of Brodmann C. The visual area is also called the striate area D. The chief source of afferent fibers to area 17 is the medial geniculate nucleus of metathalamus by way of the geniculocalcarine tract E. All of the above

Answer 1.C 2.C 3.C 4.E 5.B 6.E 7.D 8.C 9.C 10.C 11.A 12.A 13.D 14.B 15.B 16.A 17.B 18.C 19.C 20.C 21.C 22.B 23.C 24.A 25.D 26.C 27.B 28.B 29.A 30.B 31.A 32.A 33.C 34.E 35.A 36.A 37.E 38.A 39.A 40.E 41.E 42.C

周 zhōu

围 wéi

神 shén

经 jīng

系 xì

统 tǒng

Peripheral nerveous system

1. Upon examination of a patient you determine that a peripheral nerve has been lesioned. Which of the following are TRUE regarding peripheral nerves? A. They contain fibroblasts and connective tissue fibers B. When transected, their axons undergo degeneration distal to the site of injury C. They lack a blood supply D. A, B, and C are correct E. A and B are correct 2. Which of the following are associated with the PNS? A. Anterior root B. Posterior ramus C. Anterior funiculus D. Olfactory tract E. A and B 3. The phrenic nerve arises from the A. Cervical plexus B. Brachial plexus C. Lumbar plexus D. Sacral plexus E. Celiac plexus 4. Which of the following cranial nerves is NOT a mixed nerve? A. Vagus B. Trigeminal C. Glossopharyngeal

Page 63: Mid term exam(anatomy) 2012

系统解剖学习题(留学生) 63

D.Vestibulocochlear E. Facial 5. Parasympathetic cell bodies are located in each of the following gan­glions EXCEPT: A. Otic ganglion. B. Sphenopalatine ganglion C. Stellate ganglion D. Ciliary ganglion E. Submandibular ganglion 6. The main sensory nerve of the face is; A. Trigeminal n. B. Glossopharyngeal n. C. Facial n. D. Hypoglossal n. E. Accessory n. 7. All of the following muscle­nerve combination is correct EXCEPT: A. Aupraspinatus m, ­suprascapular n. B. Rhomboideus m. ­dorsal scapular n. C. Sen­atus anterior m. ­long thoracic n. D. Deltoid m. ­thoracodorsal n. E. Brachialis m. ­musculocutaneous n. 8. Autonomy: ganglia located on either side of the vertebral column are: A. Prevertebral B. Paravertebral C. Parasympathetic D. Collateral E. B and C 9. The nucleus of the solitary tract receive afferent fibers from the cranial nerve: A. The V 、 Ⅶ 、Ⅸ B. The V 、 Ⅸ 、Ⅹ C. The Ⅶ、 Ⅸ 、Ⅹ D.TheⅨ 、Ⅹ、 Ⅺ E.TheⅩ、 Ⅺ、 Ⅷ 10. The sympathetic nervous system: A, Has its preganglionic nerve cell bodies locatedintheintermediategraymatterofthe spinal cord B.Has a more restricted anatomicdistribution than does the parasympathetic nervous system C. Has its effects inhibited by secretions of the adrenal cortex D. Includes preaortic or prevertebral ganglia in the thorax E.Innervation structures located in the dermis of the skin 1l. Regarding the autonomic nervous system, the following are all ture EXCEPT: A.It lacks afferent fibers B.Its preganglionic axons arise from neurons having their cell bodies located in the central nervous system C.The white rami communicantes are made up of axons coursing from the spinal nerve into the sympathetic trunk

Page 64: Mid term exam(anatomy) 2012

系统解剖学习题(留学生) 64

D. The sympathetic ganlglia tend to be larger than the parasympathetic gangglia E. The sympathetic division is also called the thoracolumbar division 12. The sympathetic trunk: A. Exlends from the base of the skull to the tip of the coccyx B. In the abdomen lacks white rami communicantes C. In the neck gives rise to branches of the direct visceral type which innervate viscera in the thorax D.A and C are correct E.A,B and C are correct 13. The following are all true regarding the spinal nerves EXCEPT: A. The dorsal primary ramus is both motor and sensory in function B. Postganglionic sympathetic axons are present in the anterior primary rami of all spinal nerves C. The eighth pair of thoracic nerves exits between thoracic vertebrae T8 and T9 D. The thoracic nerves give rise to lateral cutaneous branches that enter the superficial fascia in the mid­axillary line E. Preganglionic sympathetic axons are present in the ventral roots of all spinal nerves 14. The following structures all receive motor innervation from the autonomic nervous system EXCEPT the: A. Tracheal musculature B. Heart musculature C. Muscle of the diaphragm ­ D. Muscle in the wall of the coronary arteries E. Muscular layers of the stomach 15. It was noted that after the injury the patient's face on the right side was flushed due to dilation of the blood vessels. The lack of vasoconstriction was due to interruption of what fibers somewhere along their course? A. Somatic afferent B. Somatic efferent C. Parasympathetic D. Sympathetic E. All of above 16. The greater splanchnic nerves: A. Consist of postganglionic sympathetic fibers B. Arise from the Ts to Tg thoracic sympathetic ganglia C. Predominately enter the abdomen through the esophageal hiatus D. Innervate only thoracic viscera E. None of the above are correct 17. Patients experiencing a heart attack commonly report pain. in the upper left extremity. The afferent nerve fibers that transmit this pain are probably in the: A.Greater splanchnic nerve B. Lesser splanchnic nerve C. Paracardicophrenic nerve D. Vagus nerve E. Sympathetic cardiac nerves 18. During the repair of an aorta that is constricted at the level of the ligamentum arteriosum,

Page 65: Mid term exam(anatomy) 2012

系统解剖学习题(留学生) 65

what nerve must be identified and preserved? A. Anterior vagal trunk B. left phrenic nerve C. left sympathetic trunk D. left pulmonary plexus E. left recurrent laryngeal nerve 19. Efferent supply to the striated muscle of the upper esophagus is by the nerves. A. Phrenic B. Sympathetic C. Thoracic visceral D. Recurrent laryngeal E. External laryngeal 20. The patient is experiencing visceral pain originating in the sigmoid colon. Nerve fibers that carry this sensation to the spinal cord pass through which of the following? A.Genital branch of the genitofemoral nerve B. Ilioinguinal nerve C. Iliohypogastric nerve D. Sympathetic trunk E. Vagus nerve 21. The sympathetic nervous system: A. Is widespread in its distribution B. Has postganglionic neurons which may have their nerve cell bodies in the sympathetic trunk C. Supplies gray rami communicans to anterior primary rami of all spinal nerves D. Irinervation abdominal viscera by branches that course through the thorax E. All of the above 22. The sympathetic nervous system: A. Has postganglionic neurons which may have their nerve cell bodies in the sympathetic trunk B. Supplies white rami communicans to anterior primary rami of all spinal nerves C. Innervation all abdominal viscera by preganglionic nerves that arise within the abdomen D. Is also called the craniosacral division of the autonomic nervous system E. None of the above 23. Which of the following is a FALSE statement about the vagus nerve? A.It is the tenth ( X ) cranial nerve B It contains preganglionic parasympathetic axons C. It contributes to part of the cardiac and pulmonary plexuses D. It passes into the abdominal region in contact with the esophagus E. It slows down the peristaltic contractions of the gut 24. All of the following statements are true regarding the celiac plexus of nerves EXCEPT: A. It is anatomically adjacent to the celiac lymph nodes B. It contains only postganglionic sympathetic nerve fibers C. It supplies innervation to the liver and gall bladder D. It supplies innervation to the first part of the duodenum E. Many of its nerve fibers course with arteries 25. The obturator nerve: A. Arises from sacral levels of the spinal cord

Page 66: Mid term exam(anatomy) 2012

系统解剖学习题(留学生) 66

B. Never enters the true pelvis on its course to the lower extremity C. Innervation the rectus femoris muscle D. Has an anterior branch that lies on the posterior surface of adductor brevis E. Is the'motor supply to the gracilis muscle 26. If a person looking inward towards their nose is unable to look down, which nerve may be injured: A. Abducens (CN VI) B. Inferior division of oculomotor ( Ⅲ ) C. Optic (Ⅱ) D. Superior division of oculomotor (Ⅲ) E. Trochlear (IV) 27. A 60­year­old male presented with a number of unusual signs and symptoms in the facial region. Among others, it was found that the right side of his face was flushed (red). Further tes­ting revealed a lack of ability to sweat in the same cutaneous region. Which nervous structures were most likely implicated in this set of clinical abnormalities? A. Cranial outflow of the CNS B. Dorsal roots of cervical nerves C. Gray rami communicantes of Ts D. Sympathetic nerve fibers E. Vagus nerves 28. The geniculate ganglion is the sensory ganglion of which nerve: A. Facial B. Glossopharyngeal C. Trigeminal D. Vagus E. Vestibulocochlear 29. A sixty­four­year old man was diagnosed with an acoustic neuroma (tumor of the Ⅷ cranialnerve) where it entered the temporal bone. What other cranial nerve ­might also be affectedsince this nerve uses the same foramen as the Ⅷ in its course? A. Abducens B. Facial C. Glossopharyngeal D. Trigeminal E. Vagus 30. A 35­year­old man was admitted to the hospital complaining of double vision (diplopia) , inability to see close objects, and blurred vision in the right eye. A verterbrobasilar angiogram re­vealed 'an aneurysm of the superior cerebellar artery close to its origin on the right side. Thedoctor attributed the symptoms to the compression of an adjacent cranial nerve by the aneurysm. The compressed nerve is the: A. Abducens (CN Ⅵ) B. Oculomotor (CN Ⅲl) C. Optic (CN Ⅱ) D. Trigeminal (CN V) E. Trochlear (CN Ⅳ) 31. While recovering from multiple dental extractions, an elderly man experienced a

Page 67: Mid term exam(anatomy) 2012

系统解剖学习题(留学生) 67

radiatingpain affecting the lower eyelid, lateral side of the nose, upper lip and over the zygomatic and temporal areas on the left side. Which nerve is involved in the patients perception of pain? A. Facial B. Ophthalmic division of trigeminal C. Gloss pharyngeal D. Mandible division of trigeminal E. Maxillary division of trigeminal 32. A 38­year­old patient complained of acute dental pain. The attending dentist found penetratingdental caries (dental decay) affecting one of the mandibular molar teeth. Which nerve would thedentist need to anesthetize to work on that tooth? A. Lingua B. Inferior alveolar C. Buccal D. Mental E. Mylohyoid 33. Which of the following suprahyoid muscles would be paralyzed if the inferior alveolar nervewere severed at its origin? A. Geniohyoid m. B. Hyoglossus m. C. Mylohyoid m. D. Stylohyoid m. E. Buccinator 34. Sympathetic fibers reach the tongue by way of the: A, Lingual nerve B. Maxillary artery. C. Hypoglossal nerve D. Lingual artery E. Glossopharyngeal nerve 35. A patient is unable to taste a piece of sugar placed on the anterior part of the tongue which cranial nerve is most likely to have a lesion? A. Facial nerve B. Glossopharyngeal nerve C. Hypoglossal nerve D. Trigeminal nerve E. Vagus nerve 36. The chorda tympani contain which component before it joins the lingual nerve? A. Preganglionic sympatheticing B. Postganglionic sympathetic C. Preganglionic parasympathetic D. Postganglionic parasympathetic E. Taste fibers to the posterior third of the tongue 37. The cell bodies of the postganglionic parasympathetic neurons innervating the sublingual gland are found in which of the following ganglia? A. Ciliary B. Optic

Page 68: Mid term exam(anatomy) 2012

系统解剖学习题(留学生) 68

C. Submandibular D. Superior cervical E. Trigeminal 38. Which of the following is a dorsal ramus of a spinal nerve? A. Supraclavicular n. B. Transverse cervical n. C. Great auricular n. D. Greater occipital n. E. Lesser occipital n. 39. Any irritation of the diaphragm (e. g. infection tumor) may create referred pain that seems (to the patient) to originate in the area atop the shoulder. This is due to the fact that the phrenic nerve shares spinal segments with what cutaneous nerve in the shoulder region? A. Dorsal scapular B. Lesser occipital nerve C. Long thoracic nerve D. Supraclavicular nerve E. Suprascapular nerve 40. A person develops a primary tumor of the thyroid gland and, among other symptoms, drooping of the eyelid and constriction of the pupil on the right side of the eye are noted. What nerve fibers have been interrupted by the tumor? A. Postganglionic parasympathetic B. Postganglionic sympathetic C. Preganglionic parasympathetic D. The carotid nerve E. The cervical sympathetic trunk 41. A 55­year­old woman has difficulty 5­wallo wing and frequently aspirates fluids while drinking. She is diagnosed as having a skull base tumor occupying the space behind the jugular foramen. Involvement of which structure is responsible for the findings? A. Ansa cervicalis R Cervical sympathetic trunk C. Accessory nerve D. Hypoglossal nerve E. Vagus 42. While doing a postoperative physical on a patient who has undergone carotid endarterectomy on the right side, it was noted that the tongue deviated toward the right when the patient was asked to point the tongue outward. What nerve crossing the carotid artery must have been injured? A. Gloss pharyngeal B. Hypoglossal C. Inferior alveolar D. Lingual E. Vagus 43. The carotid body is innervated by a branch of the: A. Glossopharyngeal nerve B. Hypoglossal nerve

Page 69: Mid term exam(anatomy) 2012

系统解剖学习题(留学生) 69

C. Accessory nerve D. Sympathetic trunk E. Vagus nerve 44. The constrictor muscles of the pharynx receive their motor nerve supply from the: A. Glossopharyngeal nerve B. Hypoglossal nerve C. Accessory nerve D. Sympathetic trunk E. Vagus nerve 45. Neurological testing of a patient reveals no cutaneous sensation on the tip of the index finger. Such a finding would indicate injury to some portion of which nerve? A. Axillary B. Median C. Musculocutaneous D. Radial E. Ulnar 46. The axillary nerve arises directly from which part of the­brachial plexus? A. Inferior trunk B. Lateral cord C. Medial cord D. Middle trunk E. Posterior cord 47. While riding a bike, a patient fell against a tree and fractured the shaft of the humerus at midlength. What nerve may be injured because of its close proximity to the injury? A. Ulnar B. Radial C. Axillary D. Medial antebrachial cutaneous E. Median 48. While recovering from an open abdominal hysterectomy (i. e. , using a midline abdominal incision to gain entry to the pelvis), a patient realizes that she has lost sensation to the skin of her anterior thigh and cannot extend her knee. Retractors holding the incision open and pressing against the posterior abdominal wall most likely caused injury to which nerve? A. Femoral B. Genitofemoral C. Iliohypogastric D. Lateral femoral cutaneous E. Obturator 49. The nerves of the lumbar plexus are arranged around specific muscles of the posterior abdominal wall. Which of these nerves lies immediately medial to the psoas major muscle. A. Femoral B. Genitofemoral C. Ilioinguinal D. Obturator E. All of the above

Page 70: Mid term exam(anatomy) 2012

系统解剖学习题(留学生) 70

50. Preganglionic parasympathetic nerve fibers within the pelvic (inferior hypogastric) plexus arise from S2­4 and enter the plexus via: A. Gray rami communicantes B. Hypogastric nerves C. Pelvic splanchnic nerves D. Sacral splanchnic nerves E. White rami communicantes 51. A caudal epidural block is a form of regional anesthetic used in childbirth. Within the sacral canal, the anesthetic agent bathes the sacral spinal nerve roots which would anesthetize all of the following nerves EXCEPT__ . A. Pelvic splanchnic B. Pudenda! C. S2 dorsal root D. Sacral splanchnic E. S2 ventral primary ramus 52. A patient presents complaining of bloodstained stools and the inability to completely empty his rectum. He also has pain along the back of his thigh and weakness of the posterior thigh muscles. Digital examination reveals a tumor in the posterolateral wall of the rectum. Pressure on what nerve plexus could cause the pain in his lower limb? A. Inferior hypogastric B. Inferior mesenteric C. Lumbar D. Sacral E. Superior hypogastric 53. The pelvic splanchnic nerves primarily carry _____ to the _____ plexus. A. Preganglionic parasympathetic __ superior hypogastric B. Preganglionic parasympathetic __ inferior hypogastric C. Postganglionic parasympathetic __ superior hypogastric D. Postganglionic sympathetic? superior hypogastric E. None of the above 54. A 45­year­old female patient complains of excessive sweating on the right side of the face and neck and in the right armpit region, where it leaves her clothing constantly stained with moisture. It is now such a terrible social embarrassment that she has become withdrawn and selfconscious. Since no medical treatment has proven effective, she is considering surgical denervation of the sweat glands in the affected areas. Which structure(s) might be removed or cut in order to alleviate her condition? A. Cervicothoracic (stellate) ganglion B. Dorsal roots of cervical nerves C. Greater thoracic splanchnic nerve D. Lumbar sympathetic trunk E. Vagus nerve 55. A cancerous growth from the body of the 9th thoracic vertebra exerts pressure anterolateral. Which structure lies in direct contact with this growth? A. Right vagus nerve

Page 71: Mid term exam(anatomy) 2012

系统解剖学习题(留学生) 71

B. Right phrenic nerve C. Right sympathetic trunk D. Right greater thoracic splanchnic nerve E. Right 9th intercostal nerve 56. You are observing a physician perform a thoracoscopic procedure. She pushes the deflated lung anteroinferior and points out a nervous structure lying across the heads of the ribs. You identify this structure as the: A Greater thoracic splanchnic nerve B. Sympathetic trunk C. Phrenic nerve D. Pulmonary plexus E. Vagus nerve 57. During a surgical procedure, a patient's right sympathetic trunk was accidentally severed just cranial to the level of spinal nerve Ti. Which function would be left intact in the affected region? A. Arrector pili muscle activity B. Dilation/constriction of blood vessels C. Sweat production D. Visceral reflex activity E. Voluntary muscle activity 58. The nerves that end on the secretory cells of the medulla of the suprarenal glands are principally: A. Preganglionic fibers from the greater thoracic splanchnic nerve B. Postganglionic fibers from the celiac plexus C. Postganglionic fibers from the aorticorenal ganglia D. Preganglionic fibers from the lesser thoracic splanchnic nerve E. Postganglionic fibers from the renal plexus

Answer

l. E 2. E 3.A 4.D 5.C 6.A 7.D 8.B 9.C 10.A 11.A 12.E 13.E 14.C 15.D 16.B 17.E 18.E 19.D 20.D 21.E 22.A 23.E 24.B 25.E 26.E 27.D 28.A 29.B 30.B 31.E 32.B 33.C 34.D 35.A 36.C 37. C 38.D 39.E 40.E 41.E 42.B 43.A 44.E 45.B 46.E 47.B 48.A 49.D 50.C 51.D 52.D 53.B 54.A 55.D 56.B 57.E 58.A

Page 72: Mid term exam(anatomy) 2012

系统解剖学习题(留学生) 72

神 shén

经 jīng

系 xì

统 tǒng

的 de

传 chuán

导 dǎo

通 tōng

路 lù

PATHWAYS OF THE NERVOUS SYSTEM

1. Which is upper motor neuron? A. Betz cells B. Neurons in anterior horn of spinal cord C. Neurons of facial nucleus D. Dorsal root ganglion E. Purkinje cells 2. Which of the following about spinothalamic pathways is FALSE? A. Lateral spinothalamic tract carries pain and temperature B. Anterior tract carries tickle, itch, crude touch and pressure C. First cell body in DRG with synapses in cord D. 2nd cell body in gray matter of cord, sends fibers to other side of cord and up through white matter to synapse in thalamus E. 3rd cell body in thalamus projects to visual cortex 3. Which of the following about deep sensory pathway of CNS is FALSE? A. Carry out proprioception, vibration, discriminative touch, weight discrimination and stereognosis B. Signals travel up spinal cord in posterior column C. Fibers cross­over in medulla to become the medial lemniscus pathway ending in thalamus D. Fibers cross­over in medulla to become the lateral lemniscus pathway ending in thalamus E. Thalamic fibers reach cortex 4. Which of the following about somatic sensory pathways is TRUE? A. First­order neuron conduct impulses to brainstem or spinal cord either spinal or cranial nerves B. Second­order neurons conducts impulses from spinal cord or brainstem to thalamus­cross over to opposite side before reaching thalamus C. Third­order neuron conducts impulses from thalamus to primary somatosensory cortex (postcentral gyrus of parietal lobe) D. Carry out superficial and deep sensory E. All are correct 5. Which of the following about muscle spindles is TRUE? A. Specialized intrafusal muscle fibers enclosed in a CT capsule and innervated by gamma motor neurons B. Stretching of the muscle stretches the muscle spindles sending sensory information back to the CNS C. Spindle sensory fiber monitor changes in muscle length D. Brain regulates muscle tone by controlling gamma fibers E. All are correct 6. Which of the following about sensory pathways to the cerebellum is TRUE? A. Major routes for proprioceptive signals to reach the cerebellum is medial spinocerebellar tract B. Major routes for proprioceptive signals to reach the cerebellum is posterior spinocerebellar tract

Page 73: Mid term exam(anatomy) 2012

系统解剖学习题(留学生) 73

C. Major routes for proprioceptive signals to reach the cerebellum is anterior spinocerebellar tract D. Subconscious information used by cerebellum for adjusting posture, balance and skilled movements E. Signal travels up to oppose side inferior cerebellar pedunle 7. Which of the following about primary motor cortex is FALSE? A. Precentral gyrus initiates voluntary movement B. Cells are called upper motor neurons C. Muscles represented unequally (according to the number of motor units) D. The fibers can’t project cerebellum E. More cortical area is needed if number of motor units in a muscle is high 8. The FALSE isAbout indirect pathways A. Initiates voluntary movment B. Complex polysynaptic circuits C. In clued basal ganglia, thalamus, cerebellum,reticular formation D. Descend in spinal cord as 5 major tracts E. All 5 tracts end upon interneurons or lower motor neurons 9. When lower motor neurons are damaged? A. Occurs flaccid paralysis B. No voluntary movement on same side as damage C. Muscle tone increase D. Muscle limp and flaccid E. No reflex actions 10. Which of the following about the pupil reflex is FALSE? A. Afferent fibers are optic nerve B. Efferent fibers is abduct nerve C. The reactive organ is sphincter of pupil D. Include direct reflex and indirect reflex E. When left optic nerve is injured, the direct reflex of left eye disappears 11. An 84­year old woman suffers a stroke, with paralysis on the right side of her body. Neurological tests show that the intracerebral hemorrhage has interrupted the blood supply to the posterior part of the frontal, the parietal and lateral portions of the temporal lobes of the left cerebral hemisphere. Which vessel was involved? A. Anterior cerebral artery B. Great cerebral vein C. Middle cerebral artery D. Middle meningeal artery E. Posterior cerebral artery

Answer 1. A 2.E 3. D 4.E 5. E 6.D 7. D 8. A 9.C 10. B 11. C

Page 74: Mid term exam(anatomy) 2012

系统解剖学习题(留学生) 74

脑 nǎo

和 hé

脊 jǐ

髓 suǐ

的 de

被 bèi

膜 mó 、血

xuè 管 guǎn

和 hé

脑 nǎo

脊 jǐ

液 yè

THE MENINGES AND BLOOD VESSELS OF BRAlN ANDSPINALCORD,

AND THE CEREBRAL SPlNAL FLUID

l. Which are associated with cerebrospinal fluid ? A. Cerebral aqueduct B. Lateral ventricle C. Choroid plexus D. Subarachnoid space E. All are correct 2. Which combinations are correct? A. Corpus callosum­diencephalon B. Corpora quadrigemina­midbrain C. Cortex­cerebellum D. All of the above E. Both B and C 3. The region space around the spinal cord that contains cerebrospinal fluid is the : A. Subdural B. Subarachnoid C. Epidural D. Supradural E. None of above 4. An obstruction in the interventricular foramen would interfere with the flow of cerebrospinal fluid into the: A. Lateral ventricle B. Third ventricIe C. Fourth ventricle D. Subarachnoid space of the spinal cord E. Epidural space 5. It is decided to image the spinal cord and spinal nerve rootlets by doing a myelogarm (injection of a radio­opaque dye into the subarachnoid space followed by a radiograph) . In order to inject the dye without injury to the spinal cord, the injection is ususally done below what vertebral level? A.L1 B.L2 C.L3 D.L4 E.L5 6. A patient is suspected of having bacterial meningitis. A lumbar puncture is performed to remove cerebroSpinalfnuid (CSF) for analysis. The fluid would be removed from the: A.Epidural space at the level of L3 B.Intervertebral foramen at the level of L2

Page 75: Mid term exam(anatomy) 2012

系统解剖学习题(留学生) 75

C.Spinal canal at level of L3 D.Subdural space at the level of L4 E.Subarachnoid space at the level of L4 7. The brain and spinal cord are enlclosed by three layers of membranes from outer to inner is: A. The dura mater,the arachnoid, the pia mater B. The dura mater,the pia mater,the arachnoid C. The pia mater,the arachnoid, the dura mater D. The pia mater, the dura mater, the arachnoid E. The arachnoid, the pia mater,the dura mater 8. Which of the following about the epidural space is WRONG? A. It is a space between the dura and the periosteum of the vertebral canal B. It contains a quantity of loose areolar tissue C. It contains lymphatic vessels and venous plexuses D. The spinal nerves on each side pass through it E. There is cerebrospinal fluid between the space 9. The cerebral dura mater: A. It is the inner periosteum of the skull B. It is in closely contain with the calvaria C. It is loosely attached at the base of skull D. Cerebrospinal fluid may leak out from the nose or ear with a fracture of the base of skull E. Sometimes, it is also the arachnoid 10. What isn't there in the cavernous sinuses? A. The internal carotid artery B. The internal jugular vein C. Abducent nerve D. Oculomotor nerve E. Trochlear nerve 11. Where are the main sources of cerebrospinal fluid?, A. The choroids plexuses B. The cerebral pia mater C. The internal carotid artery D. The internal jugular vein E. The vertebral artery 12. The direction of the blood flow in the sinuses of dura mater is as follows: A. Inferior sagittal sinus → straight transverse sinus → confluent of sinus → internal jugular vein B. Cavernous sinus → inferior petrosal sinus → internal jugular vein C. Superior sagittal sinus → inferior sagittal sinus → inferior petrosal sinus → internal jugular vein D. Sigmoid sinus → transverse sinus → confluent of sinus → internal jugular vein E. Straight sinus → superior petrosal sinus →internal jugular vein 13. The anterior cerebral artery from: A. The middle cerebral artery B. The internal carotid artery C. The vertebral artery

Page 76: Mid term exam(anatomy) 2012

系统解剖学习题(留学生) 76

D. The facial artery E. The anterior communicating artery 14. Which is correct about the middle cerebral artery? A. It supply little of the dorsolateral surface of the cerebral hemisphere B. It arises from the subclavian artery C. It formed the cerebral arterial circle D. It run horizontally to the lateral sulcus E. It down to the lateral sulcus 15. Which is correct about the brain barrier? A. Its function is to stable the environment surrounding the neurons B. It composed of four parts C. It is just in between blood and neuron in the brain and spinal cord D. It consists the capillary endothelium homogenous basement and neuroglial membrane E. All areas in brain have a blood­brain barrier 16. Which is wrong about the vein of brain ? A. The cerebral veins do not run together with the arteries B. It divided into superficial and deep groups C. The superficial vein empty into the adjacent of the pia mater D. The superficial vein drain the blood from cerebral cortex E. The deep vein empty into the straight sinus 17. Concerning the cranial cavity : A. The trigeminal nerve is motor to the muscles of mastication B. The accessory nerve supplies the trapezius muscle C. Infection in the inferior orbital region may spread to the cavernous sinus D. The circulus arteries connects the internal and external carotid arteries E. The internal carotid artery and the trochlear nerve run through the cavernous sinus 18. An infant was diagnosed as having hydrocephalus. It was determined that there was a blockagein the ventricular system of the baby’s brain between the third and fourth ventricles. The blockagetherefore must have involved: A. Central canal B. Cerebral aqueduct C. Foramen of Luschka (lateral foramen) D. Foramen of Magendie ( medial foramen) E. Intervenricular foramen 19. While riding her bicycle on campus without a helmet a student is hit by a car and falls, hitting her head on the pavement. She is brought to the Emergency Room in an unconscious state with signs of a slosed head injury. Tests reveal blood in her cerebrospinal fluid taken from a spinal tap. Diagnosis is of torn cerebral veins as they pass from the brain to the superior sagittal sinus. From which of the following was the bloody fluid taken? A. Cavernous sinus B. Epidural space C. Subarachnoid space D. Subdural space E. Vertebral venous plexus 20. Blockage of the flow of cerebrospinal fluid (CSF) within the cerebral aqueduct(of Sylvius)

Page 77: Mid term exam(anatomy) 2012

系统解剖学习题(留学生) 77

normally would result in the enlargement of all of the following ventricular spaces except the: A. Fourth ventricle B. Interventricular foramen(of Monro) C. Lateral ventricle D. Third ventricle E. All of the above 21. During childbirth, an excessive anteroposterior compression of the head may tear the anterior attachment of flax cerebri from the tentorium cerebelli. The bleeding that follows is likely to be from which of the following venous sinuses? A. Occipital sinus B. Sigmoid sinus C. Straight sinus D. Superior sagittal sinus E. Transverse sinus 22. The inferior sagittal sinus is found in the free edge of what structure? A. Diaphragma sellae B. Falx cerebelli C. Falx cerebri D. Filum terminal E. Tentorium cerebelli 23.A 60­year­old man with a previous history of right carotid endarterectomy comes to his physician complaining of light­headedness and dizziness whenever he uses his right hand vigorously. He is diagnosed as having subclavian steal syndrome due to an atherosclerosis plaque at the point where his subclavian artery branches from the brachiocephalic trunk. The cerebral insufficiency is the result of blood being stole from which artery? A. External carotid B. Internal carotid C. Middle cerebral D. Thyrocervical trunk E. Vertebral 24.Because of their structure and interconnections, which veins are especially important in the metastatic spread of cancer? A. Basilic B. Cephalic C. Dorsal scapular D. Internal vertebral venous plexus E. Transverse cervical

Answer 1.E 2.E 3.B 4.B 5.D 6.E 7.A 8.E 9.D 10.B 11.A 12.B 13.B 14.D 15.A 16.C 17.C 18.B 19.C 20.A 21.C 22.C 23.E 24.D

Page 78: Mid term exam(anatomy) 2012

系统解剖学习题(留学生) 78

The nervous system

Define the following Terms 1)Neuron 2)reflex 3) reflex arc 4) gray matter 5) white matter 6) nucleus 7) ganglion 8) nerve 9) fasciculus 10) Cortex 11) preganglionic fiber 12) postganglionic fiber

Essay question 1. Describe the location and external features of the spinal cord, the relation between the spinal segments and the vertebrae. 2. Describe the locations and the functions of the main nuclei of gray matter in spinal cord. 3. Describe the location and parts of brain and the composition of brain stem. 4. Describe the main fissures, sulci and gyri on the surface of the cerebral hemispheres and the lobes of cerebrum. 5. Describe the main functional area of the cerebral cortex. 6. Describe the location, parts and the passing tracts/fibers of the internal capsule. 7. Describe the name and their origin of the cranial nerves on the surface of brain according to their Roman numbers. 8. Describe passage in the base of the skull, types and distributions of the cranial nerves. 9. Describe the deep sensory pathway of trunk and limbs. 10. Describe the superficial sensory pathway of trunk and limbs. 11.Describe the visual pathway and the pupillary light reflexes pathway. 12. Describe the produce and circulative pathway of the cerebral spinal fluid. 13.Which nerve is easily to be damaged while the fracture occurred at the surgical neck of humerus,the middle part of humerus and the fibular neck separately?What are the clinical symptoms after the nerves were injured separately? Why? 14.What are the clinical symptoms under the condition of hemorrhage of internal capsule? Why?

Page 79: Mid term exam(anatomy) 2012

系统解剖学习题(留学生) 79

(一 yī

) 单 项 dānxiàng

选 择 题 xuǎnzétí

1. 在 zài

前 臂 qiánbì

两 liǎng

点 diǎn

中 zhōng

, 近 jìn

肘 关 节 zhǒuguānjié

的 de

一 点 yīdiǎn

为 wéi

( C ) A、Medial B、Lateral C、Proximal D、Proximal E、Proximal

2.在 zài

躯体 qūtǐ

两 liǎng

点 diǎn

中 zhōng

,近 jìn

正 中 zhèngzhōng

面 的 miànde

一 点 yīdiǎn

为 wéi

( A )

A、内 侧 nèicè

B、 外 wài

侧 cè

C、近 jìn

侧 cè

D、 远 yuǎn

侧 E、内 nèi

3. 内 脏 nèizāng

( C )

A、 包 括 bāokuò

消 化 xiāohuà

、呼吸 hūxī

和 hé

泌 尿 mìniào

3个 gè

系 统 xìtǒng

B、 全 部 quánbù

位 于 wèiyú

胸 xiōng

、 腹 腔 fùqiāng

内 nèi

C、各 gè

系 统 xìtǒng

都 dōu

借 jiè

孔 kǒng

、 道 dào

直 接 zhíjiē

或 huò

间 接 jiànjiē

与 yǔ

外 界 wàijiè

相 通 xiāngtōng

D、 心 xīn

是 shì

内 脏 nèizāng

器 官 qìguān

E、脾 pí

也 yě

是 shì

内 脏 nèizāng

器 官 qìguān

4.不 bù

属 于 shǔyú

内 脏 nèizāng

的 de

系 统 xìtǒng

是 shì

( B )

A、 泌 尿 系 统 mìniàoxìtǒng

B、 脉 管 màiguǎn

系 统 xìtǒng

C、 生 殖 系 统 shēngzhíxìtǒng

D、呼 吸 系 统 hūxīxìtǒng

E、 消 化 系 统 xiāohuàxìtǒng

5. 属 shǔ

中 空 zhōngkōng

性 器 官 xìngqìguān

的 de

是 shì

( E )

A、 肾 shèn

、 输 尿 管 shūniàoguǎn

、 膀 胱 pángguāng

B、 气 管 qìguǎn

、 肺 fèi

C、 胃 wèi

、 肝 gān

D、 盲 肠 mángcháng

、 阑 尾 lánwěi

、胰 yí

E、 输 卵 管 shūluǎnguǎn

、 子 宫 zǐgōng

6. 属 shǔ

实 质 shízhì

性 器 官 xìngqìguān

的 de

是 shì

( A )

A、 肝 gān

、胰 yí

B、 主 zhǔ

支 气 管 zhīqìguǎn

、 肺 fèi

C、 肾 shèn

、 输 尿 管 shūniàoguǎn

D、 前 列 腺 qiánlièxiàn

、 输 精 管 shūjīngguǎn

E、 卵 巢 luǎncháo

、 子 宫 zǐgōng

7. 有 yǒu

“门” m é n

的 de

器 官 qìguān

是 shì

( B )

A、 横 结 肠 héngjiécháng

B、 肾 shèn

C、 输 卵 管 shūluǎnguǎn

D、 气 管 qìguǎn

E、 胃 wèi

8.不 bù

属 于 shǔyú

Viscera的 de

System 是 shì

( D ) A、Digestive system B、Urinary system C、Respiratory system D、Cardiovascular system E、Reproductive system

Page 80: Mid term exam(anatomy) 2012

系统解剖学习题(留学生) 80

9. 属 shǔ

于 yú

中 zhōng

空 kōng

性 xìng

器 qì

官 guān

的 de

是 shì

( B ) A、Liver B、Colon C、Lung D、Kidney E、Pancreas

10.属 于 shǔyú

实 质 shízhì

性 器 官 xìngqìguān

的 de

是 shì

( E ) A、Urinary bladder B、Uterus C、Uterine tube D、Esophagus E、Liver

11. 肺 循 环 fèixúnhuán

( B )

A、 只 zhī

通 过 tōngguò

肺 fèi

, 主 要 zhǔyào

使 shǐ

动 脉 血 dòngmàixuè

转 变 成 zhuǎnbiànchéng

静 脉 血 jìngmàixuè

B、 血 xuè

由 yóu

右 心 室 yòuxīnshì

搏 bó

出 chū

C、 血 xuè

由 yóu

肺 静 脉 fèijìngmài

回 huí

左 心 室 zuǒxīnshì

D、 肺 动 脉 干 fèidòngmàigàn

内 流 nèiliú

着 zháo

动 脉 血 dòngmàixuè

E、 肺 静 脉 fèijìngmài

导 dǎo

着 zháo

静 脉 血 jìngmàixuè

回 huí

心 xīn

12. 体 循 环 tǐxúnhuán

( E )

A、以 yǐ

静 脉 血 jìngmàixuè

起 qǐ

于 yú

左 心 室 zuǒxīnshì

B、以 yǐ

动 脉 血 dòngmàixuè

终 于 zhōngyú

右 心 房 yòuxīnfáng

C、 循 环 xúnhuán

与 yǔ

神 经 shénjīng

体 液 调 节 tǐyètiáojié

无 关 wúguān

D、 循 环 xúnhuán

路 程 lùchéng

短 duǎn

E、以 yǐ

动 脉 血 dòngmàixuè

滋 养 zīyǎng

全 身 quánshēn

各 gè

部 bù

(二 èr

) 多 项 duōxiàng

选 择 题 xuǎnzétí

1.体 表 tǐbiǎo

可 kě

触 及 chùjí

的 de

骨 gǔ

性 xìng

标 志 biāozhì

是 shì

( ABDE )

A、乳突 rǔtū

B、剑 突 jiàntū

C、 横 突 héngtū

D、 胫 骨 粗 隆 jìnggǔcūlóng

E、坐 骨 zuògǔ

结节 jiéjié

2. 关 节 guānjié

的 de

基 本 jīběn

结 构 jiégòu

包 括 bāokuò

( BDE )

A、 韧 带 rèndài

B、 关 节 面 guānjiémiàn

C、 关 节 guānjié

内 nèi

软 骨 ruǎngǔ

D、 关 节 腔 guānjiéqiāng

E、 关 节 囊 guānjiénáng

3. 腔 qiāng

内 含 nèihán

有 yǒu

关 节 盘 guānjiépán

的 de

关 节 guānjié

是 shì

( DC )

A、 膝 关 节 xīguānjié

B、 肩 关 节 jiānguānjié

C、下 颌 xiàhé

关 节 guānjié

D、 胸 锁 关 节 xiōngsuǒguānjié

E、 髋 关 节 kuānguānjié

4. 跨 kuà

过 guò

两 liǎng

个 gè

以 yǐ

上 shàng

关 guān

节 jié

的 de

肌 jī

肉 ròu

是 shì

( BCDE ) A、deltoid muscle B、biceps brachii muscle

Page 81: Mid term exam(anatomy) 2012

系统解剖学习题(留学生) 81

C、quadriceps femoris muscle D、triceps surae muscle E、erector spine muscle

5. 外 形 wàixíng

显 露 xiǎnlù

且 qiě

可 kě

触 及 chùjí

的 de

肌 jī ( 腱 jiàn

) 是 shì

( ABCDE )

A、 小 腿 三 头 肌 xiǎotuǐsāntóujī

B、 掌 长 肌 zhǎngzhǎngjī

腱 jiàn

C、背 阔 肌 bèikuòjī

D、斜 方 肌 xiéfāngjī

E、 肱 gōng

三 sān

头 tóu

肌 jī

6.具 jù

有 yǒu

屈 qū

光 guāng

作 zuò

用 yòng

者 zhě

是 shì

( ABDE )

A、Cornea B、 瞳 tóng

孔 kǒng

C、Iris D、Lens E、 房 fáng

水 shuǐ

7.属 于 shǔyú

膜 迷 路 mómílù

者 zhě

是 shì

( BCD )

A、 前 庭 qiántíng

B、 椭 圆 囊 tuǒyuánnáng

C、 球 囊 qiúnáng

D、 蜗 管 wōguǎn

E、耳蜗 ěrwō

8.单 个 dāngè

的 de

内 分 泌 腺 nèifēnmìxiàn

是 shì

( ABC )

A、 垂 体 chuítǐ

B、 松 果 体 sōngguǒtǐ

C、 甲 状 腺 jiǎzhuàngxiàn

D、 甲 状 旁 腺 jiǎzhuàngpángxiàn

E、 肾 上 腺 shènshàngxiàn

9. 有 yǒu

语 言 yǔyán

中 枢 zhōngshū

的 de

脑 回 nǎohuí

是 shì

(ABCD)

A、 角 回 jiǎohuí

B、 颞 上 回 nièshànghuí

C、 额 中 回 ézhōnghuí

D、额下回 éxiàhuí

E、 颞 横 回 nièhénghuí

10. 脉 络 丛 màiluòcóng

存 在 cúnzài

于 yú

( ACD )

A、侧 脑 室 cènǎoshì

B、 中 脑 水 管 zhōngnǎoshuǐguǎn

C、第 dì

Ⅲ 脑 室 nǎoshì

D、第 dì

Ⅳ 脑 室 nǎoshì

E、 蛛 网 膜 下 隙 zhūwǎngmóxiàxì

11.属 于 shǔyú

基底 jīdǐ

神 经 核 shénjīnghé

的 de

是 shì

( ABDE )

A、 尾 状 核 wěizhuànghé

B、 豆 状 核 dòuzhuànghé

C、 齿 状 核 chǐzhuànghé

D、 杏 仁 核 xìngrénhé

E、 屏 状 核 píngzhuànghé

12. 接 受 jiēshòu

手 shǒu

部 bù

皮肤 pífū

感 觉 gǎnjiào

的 de

神 经 shénjīng

有 yǒu

(ABD)

A、 正 中 神 经 zhèngzhōngshénjīng

B、 尺 神 经 chǐshénjīng

C、 肌 皮 神 经 jīpíshénjīng

D、 桡 神 经 ráoshénjīng

浅 支 qiǎnzhī

E、 桡 神 经 ráoshénjīng

深 支 shēnzhī

Page 82: Mid term exam(anatomy) 2012

系统解剖学习题(留学生) 82

13. 经 jīng

眶 上 裂 kuàngshàngliè

出 chū

(入 rù )颅 lú

的 de

脑 神 经 nǎoshénjīng

有 yǒu

( ABC )

A、 动 眼 神 经 dòngyǎnshénjīng

B、 滑 车 神 经 huáchēshénjīng

C、 展 神 经 zhǎnshénjīng

D、 上 颌 神 经 shànghéshénjīng

E、 下 颌 神 经 xiàhéshénjīng

14. 支 配 zhīpèi

头肌 tóujī

运 动 yùndòng

的 de

脑 神 经 nǎoshénjīng

有 yǒu

(BD)

A、 上 颌 神 经 shànghéshénjīng

B、 面 神 经 miànshénjīng

C、 展 神 经 zhǎnshénjīng

D、 下 颌 神 经 xiàhéshénjīng

E、 滑 车 神 经 huáchēshénjīng